Lewis - Cardio and Vascular

अब Quizwiz के साथ अपने होमवर्क और परीक्षाओं को एस करें!

A patient in the outpatient clinic has a new diagnosis of peripheral artery disease (PAD). Which group of drugs will the nurse plan to include when teaching about PAD management? a. Statins c. Thrombolytics b. Antibiotics d. Anticoagulants

ANS A Research indicates that statin use by patients with PAD improves multiple outcomes. There is no research that supports the use of the other drug categories in PAD. DIF: Cognitive Level: Apply (application) REF: 805 TOP: Nursing Process: Planning MSC: NCLEX: Physiological Integrity

When analyzing an electrocardiographic (ECG) rhythm strip of a patient with a regular heart rhythm, the nurse counts 30 small blocks from one R wave to the next. The nurse calculates the patient's heart rate as ____.

ANS: 50 There are 1500 small blocks in a minute, and the nurse will divide 1500 by 30.

Which assessment finding in a patient admitted with acute decompensated heart failure (ADHF) requires the most immediate action by the nurse? a. Oxygen saturation of 88% b. Weight gain of 1 kg (2.2 lb) c. Heart rate of 106 beats/minute d. Urine output of 50 mL over 2 hours

ANS: A A decrease in oxygen saturation to less than 92% indicates hypoxemia. The nurse should administer supplemental oxygen immediately to the patient. An increase in apical pulse rate, 1-kg weight gain, and decreases in urine output also indicate worsening heart failure and require nursing actions, but the low oxygen saturation rate requires the most immediate nursing action

25. The nurse is caring for a patient who was admitted to the coronary care unit following an acute myocardial infarction (AMI) and percutaneous coronary intervention the previous day. Teaching for this patient would include a. when cardiac rehabilitation will begin. b. the typical emotional responses to AMI. c. information regarding discharge medications. d. the pathophysiology of coronary artery disease.

ANS: A Early after an AMI, the patient will want to know when resumption of usual activities can be expected. At this time, the patient's anxiety level or denial will interfere with good understanding of complex information such as the pathophysiology of coronary artery disease (CAD). Teaching about discharge medications should be done closer to discharge. The nurse should support the patient by decreasing anxiety rather than discussing the typical emotional responses to myocardial infarction (MI).

To determine whether there is a delay in impulse conduction through the atria, the nurse will measure the duration of the patient's a. P wave. b. Q wave. c. P-R interval. d. QRS complex.

ANS: A The P wave represents the depolarization of the atria. The P-R interval represents depolarization of the atria, atrioventricular (AV) node, bundle of His, bundle branches, and the Purkinje fibers. The QRS represents ventricular depolarization. The Q wave is the first negative deflection following the P wave and should be narrow and short

A patient's cardiac monitor shows a pattern of undulations of varying contours and amplitude with no measurable ECG pattern. The patient is unconscious and pulseless. Which action should the nurse take first? a. Perform immediate defibrillation. b. Give epinephrine (Adrenalin) IV. c. Prepare for endotracheal intubation. d. Give ventilations with a bag-valve-mask device.

ANS: A The patient's rhythm and assessment indicate ventricular fibrillation and cardiac arrest; the initial action should be to defibrillate. If a defibrillator is not immediately available or is unsuccessful in converting the patient to a better rhythm, the other actions may be appropriate

39. To improve the physical activity level for a mildly obese 71-year-old patient, which action should the nurse plan to take? a. Stress that weight loss is a major benefit of increased exercise. b. Determine what kind of physical activities the patient usually enjoys. c. Tell the patient that older adults should exercise for no more than 20 minutes at a time. d. Teach the patient to include a short warm-up period at the beginning of physical activity.

ANS: B Because patients are more likely to continue physical activities that they already enjoy, the nurse will plan to ask the patient about preferred activities. The goal for older adults is 30 minutes of moderate activity on most days. Older adults should plan for a longer warm-up period. Benefits of exercises, such as improved activity tolerance, should be emphasized rather than aiming for significant weight loss in older mildly obese adults.

A 53-year-old patient with Stage D heart failure and type 2 diabetes asks the nurse whether heart transplant is a possible therapy. Which response by the nurse is most appropriate? a. "Because you have diabetes, you would not be a candidate for a heart transplant." b. "The choice of a patient for a heart transplant depends on many different factors." c. "Your heart failure has not reached the stage in which heart transplants are needed." d. "People who have heart transplants are at risk for multiple complications after surgery."

ANS: B Indications for a heart transplant include end-stage heart failure (Stage D), but other factors such as coping skills, family support, and patient motivation to follow the rigorous posttransplant regimen are also considered. Diabetic patients who have well-controlled blood glucose levels may be candidates for heart transplant. Although heart transplants can be associated with many complications, this response does not address the patient's question

A patient has ST segment changes that support an acute inferior wall myocardial infarction. Which lead would be best for monitoring the patient? a. I b. II c. V2 d. V6

ANS: B Leads II, III, and AVF reflect the inferior area of the heart and the ST segment changes. Lead II will best capture any electrocardiographic (ECG) changes that indicate further damage to the myocardium. The other leads do not reflect the inferior part of the myocardial wall and will not provide data about further ischemic changes in that area

31. The nurse is assessing a patient with myocarditis before giving the scheduled dose of digoxin (Lanoxin). Which finding is most important for the nurse to communicate to the health care provider? a. Leukocytosis c. Generalized myalgia b. Irregular pulse d. Complaint of fatigue

ANS: B Myocarditis predisposes the heart to digoxin-associated dysrhythmias and toxicity. The other findings are common symptoms of myocarditis and there is no urgent need to report these. DIF: Cognitive Level: Analyze (analysis) REF: 787 OBJ: Special Questions: Prioritization TOP: Nursing Process: Assessment MSC: NCLEX: Physiological Integrity

2. During the assessment of a young adult patient with infective endocarditis (IE), the nurse would expect to find a. substernal chest pressure. c. a pruritic rash on the chest. b. a new regurgitant murmur. d. involuntary muscle movement.

ANS: B New regurgitant murmurs occur in IE because vegetations on the valves prevent valve closure. Substernal chest discomfort, rashes, and involuntary muscle movement are clinical manifestations of other cardiac disorders such as angina and rheumatic fever. DIF: Cognitive Level: Understand (comprehension) REF: 790 TOP: Nursing Process: Assessment MSC: NCLEX: Physiological Integrity

10. The nurse will suspect that the patient with stable angina is experiencing a side effect of the prescribed metoprolol (Lopressor) if the a. patient is restless and agitated. b. blood pressure is 90/54 mm Hg. c. patient complains about feeling anxious. d. cardiac monitor shows a heart rate of 61 beats/minute.

ANS: B Patients taking β-adrenergic blockers should be monitored for hypotension and bradycardia. Because this class of medication inhibits the sympathetic nervous system, restlessness, agitation, hypertension, and anxiety will not be side effects.

9. Diltiazem (Cardizem) is ordered for a patient with newly diagnosed Prinzmetal's (variant) angina. When teaching the patient, the nurse will include the information that diltiazem will a. reduce heart palpitations. b. decrease spasm of the coronary arteries. c. increase the force of the heart contractions. d. help prevent plaque from forming in the coronary arteries.

ANS: B Prinzmetal's angina is caused by coronary artery spasm. Calcium channel blockers (e.g., diltiazem, amlodipine [Norvasc

9. The nurse is admitting a patient with possible rheumatic fever. Which question on the admission health history focuses on a pertinent risk factor for rheumatic fever? a. "Do you use any illegal IV drugs?" b. "Have you had a recent sore throat?" c. "Have you injured your chest in the last few weeks?" d. "Do you have a family history of congenital heart disease?"

ANS: B Rheumatic fever occurs as a result of an abnormal immune response to a streptococcal infection. Although illicit IV drug use should be discussed with the patient before discharge, it is not a risk factor for rheumatic fever, and it would not be as pertinent when admitting the patient. Family history is not a risk factor for rheumatic fever. Chest injury would cause musculoskeletal chest pain rather than rheumatic fever. DIF: Cognitive Level: Apply (application) REF: 788 TOP: Nursing Process: Assessment MSC: NCLEX: Physiological Integrity

The nurse notes that a patient's cardiac monitor shows that every other beat is earlier than expected, has no visible P wave, and has a QRS complex that is wide and bizarre in shape. How will the nurse document the rhythm? a. Ventricular couplets b. Ventricular bigeminy c. Ventricular R-on-T phenomenon d. Multifocal premature ventricular contractions

ANS: B Ventricular bigeminy describes a rhythm in which every other QRS complex is wide and bizarre looking. Pairs of wide QRS complexes are described as ventricular couplets. There is no indication that the premature ventricular contractions (PVCs) are multifocal or that the R-on-T phenomenon is occurring

An older patient with chronic atrial fibrillation develops sudden severe pain, pulselessness, pallor, and coolness in the right leg. The nurse should notify the health care provider and immediately a. apply a compression stocking to the leg. b. elevate the leg above the level of the heart. c. assist the patient in gently exercising the leg. d. keep the patient in bed in the supine position.

ANS D The patient's history and clinical manifestations are consistent with acute arterial occlusion, and resting the leg will decrease the O2 demand of the tissues and minimize ischemic damage until circulation can be restored. Elevating the leg or applying an elastic wrap will further compromise blood flow to the leg. Exercise will increase oxygen demand for the tissues of the leg. DIF: Cognitive Level: Apply (application) REF: 808 TOP: Nursing Process: Implementation MSC: NCLEX: Physiological Integrity

34. Which action could the nurse delegate to unlicensed assistive personnel (UAP) trained as electrocardiogram (ECG) technicians working on the cardiac unit? a. Select the best lead for monitoring a patient with an admission diagnosis of Dressler syndrome. b. Obtain a list of herbal medications used at home while admitting a new patient with pericarditis. c. Teach about the need to monitor the weight daily for a patient who has hypertrophic cardiomyopathy. d. Watch the heart monitor for changes in rhythm while a patient who had a valve replacement ambulates.

ANS: D Under the supervision of registered nurses (RNs), UAPs check the patient's cardiac monitor and obtain information about changes in heart rate and rhythm with exercise. Teaching and obtaining information about home medications (prescribed or complementary) and selecting the best leads for monitoring patients require more critical thinking and should be done by the RN. DIF: Cognitive Level: Apply (application) REF: 782 OBJ: Special Questions: Delegation TOP: Nursing Process: Planning MSC: NCLEX: Safe and Effective Care Environment

22. The nurse will plan discharge teaching about prophylactic antibiotics before dental procedures for which patient? a. Patient admitted with a large acute myocardial infarction b. Patient being discharged after an exacerbation of heart failure c. Patient who had a mitral valve replacement with a mechanical valve d. Patient being treated for rheumatic fever after a streptococcal infection

ANS: C Current American Heart Association guidelines recommend the use of prophylactic antibiotics before dental procedures for patients with prosthetic valves to prevent infective endocarditis (IE). The other patients are not at risk for IE. DIF: Cognitive Level: Apply (application) REF: 783 TOP: Nursing Process: Planning MSC: NCLEX: Physiological Integrity

19. When caring for a patient with infective endocarditis of the tricuspid valve, the nurse should monitor the patient for the development of a. flank pain. c. shortness of breath. b. splenomegaly. d. mental status changes.

ANS: C Embolization from the tricuspid valve would cause symptoms of pulmonary embolus. Flank pain, changes in mental status, and splenomegaly would be associated with embolization from the left-sided valves. DIF: Cognitive Level: Apply (application) REF: 792 TOP: Nursing Process: Planning MSC: NCLEX: Physiological Integrity

The nurse plans discharge teaching for a patient with chronic heart failure who has prescriptions for digoxin (Lanoxin) and hydrochlorothiazide (HydroDIURIL). Appropriate instructions for the patient include a. limit dietary sources of potassium. b. take the hydrochlorothiazide before bedtime. c. notify the health care provider if nausea develops. d. skip the digoxin if the pulse is below 60 beats/minute.

ANS: C Nausea is an indication of digoxin toxicity and should be reported so that the provider can assess the patient for toxicity and adjust the digoxin dose, if necessary. The patient will need to include potassium-containing foods in the diet to avoid hypokalemia. Patients should be taught to check their pulse daily before taking the digoxin and if the pulse is less than 60, to call their provider before taking the digoxin. Diuretics should be taken early in the day to avoid sleep disruption

18. A patient recovering from heart surgery develops pericarditis and complains of level 6 (0 to 10 scale) chest pain with deep breathing. Which prescribed PRN medication will be the most appropriate for the nurse to give? a. Fentanyl 1 mg IV c. Oral ibuprofen (Motrin) 600 mg b. IV morphine sulfate 4 mg d. Oral acetaminophen (Tylenol) 650 mg

ANS: C The pain associated with pericarditis is caused by inflammation, so nonsteroidal antiinflammatory drugs (e.g., ibuprofen) are most effective. Opioid analgesics and acetaminophen are not very effective for the pain associated with pericarditis. DIF: Cognitive Level: Analyze (analysis) REF: 784 TOP: Nursing Process: Implementation MSC: NCLEX: Physiological Integrity

30. Which electrocardiographic (ECG) change is most important for the nurse to report to the health care provider when caring for a patient with chest pain? a. Inverted P wave b. Sinus tachycardia c. ST-segment elevation d. First-degree atrioventricular block

ANS: C The patient is likely to be experiencing an ST-segment-elevation myocardial infarction (STEMI). Immediate therapy with percutaneous coronary intervention (PCI) or thrombolytic medication is indicated to minimize myocardial damage. The other ECG changes may also suggest a need for therapy, but not as rapidly.

37. A patient who has chest pain is admitted to the emergency department (ED) and all of the following are ordered. Which one should the nurse arrange to be completed first? a. Chest x-ray b. Troponin level c. Electrocardiogram (ECG) d. Insertion of a peripheral IV

ANS: C The priority for the patient is to determine whether an acute myocardial infarction (AMI) is occurring so that reperfusion therapy can begin as quickly as possible. ECG changes occur very rapidly after coronary artery occlusion, and an ECG should be obtained as soon as possible. Troponin levels will increase after about 3 hours. Data from the chest x-ray may impact the patient's care but are not helpful in determining whether the patient is experiencing a myocardial infarction (MI). Peripheral access will be needed but not before the ECG.

) are a first-line therapy for this type of angina. Lipid-lowering drugs help reduce atherosclerosis (i.e., plaque formation), and β-adrenergic blockers decrease sympathetic stimulation of the heart (i.e., palpitations). Medications or activities that increase myocardial contractility will increase the incidence of angina by increasing oxygen demand.

...

When assessing a patient with possible peripheral artery disease (PAD), the nurse obtains a brachial blood pressure (BP) of 147/82 mm Hg and an ankle pressure of 112/74 mm Hg. The nurse calculates the patient's ankle-brachial index (ABI) as ________ (round up to the nearest hundredth).

ANS 0.76 The ABI is calculated by dividing the ankle systolic BP by the brachial systolic BP. DIF: Cognitive Level: Apply (application) REF: 805 TOP: Nursing Process: Implementation MSC: NCLEX: Physiological Integrity

21. Which assessment finding in a patient admitted with acute decompensated heart failure (ADHF) requires the most immediate action by the nurse? a. O2 saturation of 88% b. Weight gain of 1 kg (2.2 lb) c. Heart rate of 106 beats/min d. Urine output of 50 mL over 2 hours

ANS A A decrease in O2 saturation to less than 92% indicates hypoxemia, and the nurse should start supplemental O2 immediately. An increase in apical pulse rate, 1-kg weight gain, and decreases in urine output also indicate worsening heart failure and require nursing actions, but the low O2 saturation rate requires the most immediate nursing action. DIF: Cognitive Level: Analyze (analysis) REF: 741 OBJ: Special Questions: Prioritization TOP: Nursing Process: Assessment MSC: NCLEX: Physiological Integrity

The nurse is developing a discharge teaching plan for a patient diagnosed with thromboangiitis obliterans (Buerger's disease). Which expected outcome has the highest priority for this patient? a. Cessation of all tobacco use b. Control of serum lipid levels c. Maintenance of appropriate weight d. Demonstration of meticulous foot care

ANS A Absolute cessation of nicotine use is needed to reduce the risk for amputation in patients with Buerger's disease. Other therapies have limited success in treatment of this disease. DIF: Cognitive Level: Analyze (analysis) REF: 809 OBJ: Special Questions: Prioritization TOP: Nursing Process: Planning MSC: NCLEX: Physiological Integrity

Which patient statement to the nurse is most consistent with the diagnosis of venous insufficiency? a. "I can't get my shoes on at the end of the day." b. "I can't ever seem to get my feet warm enough." c. "I have burning leg pains after I walk two blocks." d. "I wake up during the night because my legs hurt."

ANS A Because the edema associated with venous insufficiency increases when the patient has been standing, shoes will feel tighter at the end of the day. The other patient statements are characteristic of peripheral artery disease. DIF: Cognitive Level: Apply (application) REF: 826 TOP: Nursing Process: Assessment MSC: NCLEX: Physiological Integrity

An older patient with a history of an abdominal aortic aneurysm arrives at the emergency department (ED) with severe back pain and absent pedal pulses. Which action should the nurse take first? a. Check the blood pressure. b. Draw blood for laboratory testing. c. Assess for the presence of an abdominal bruit. d. Determine any family history of heart disease.

ANS A Because the patient appears to be experiencing aortic dissection, the nurse's first action should be to determine the hemodynamic status by assessing blood pressure. The other actions may also be done, but they will not provide information to determine what interventions are needed immediately. DIF: Cognitive Level: Analyze (analysis) REF: 814 OBJ: Special Questions: Prioritization TOP: Nursing Process: Implementation MSC: NCLEX: Physiological Integrity

When evaluating the discharge teaching for a patient with chronic peripheral artery disease (PAD), the nurse determines a need for further instruction when the patient says, "I will a. use a heating pad on my feet at night to increase the circulation." b. buy some loose clothes that do not bind across my legs or waist." c. walk to the point of pain, rest, and walk again for at least 30 minutes 3 times a week." d. change my position every hour and avoid long periods of sitting with my legs crossed."

ANS A Because the patient has impaired circulation and sensation to the feet, the use of a heating pad could lead to burns. The other patient statements are correct and indicate that teaching has been successful. DIF: Cognitive Level: Apply (application) REF: 804 TOP: Nursing Process: Evaluation MSC: NCLEX: Physiological Integrity

The nurse is caring for a patient with critical limb ischemia who has just arrived on the nursing unit after having percutaneous transluminal balloon angioplasty. Which action should the nurse perform first? a. Obtain vital signs. c. Assess pedal pulses. b. Teach wound care. d. Check the wound site.

ANS A Bleeding is a possible complication after catheterization of the femoral artery, so the nurse's first action should be to assess for changes in vital signs that might indicate hemorrhage. The other actions are also appropriate but can be done after determining that bleeding is not occurring. DIF: Cognitive Level: Analyze (analysis) REF: 804 OBJ: Special Questions: Prioritization TOP: Nursing Process: Implementation MSC: NCLEX: Physiological Integrity

7. The nurse working on the heart failure unit knows that teaching an older female patient with newly diagnosed heart failure is effective when the patient states that a. she will take furosemide (Lasix) every day at bedtime. b. the nitroglycerin patch is to be used when chest pain develops. c. she will call the clinic if her weight goes up 3 pounds in 1 week. d. an additional pillow can help her sleep if she is short of breath at night.

ANS C Teaching for a patient with heart failure includes information about the need to weigh daily and notify the health care provider about an increase of 3 lb in 2 days or 3 to 5 lb in a week. Nitroglycerin patches are used primarily to reduce preload (not to prevent chest pain) in patients with heart failure and should be used daily, not on an "as needed" basis. Diuretics should be taken earlier in the day to avoid nocturia and sleep disturbance. The patient should call the clinic if increased orthopnea develops rather than just compensating by further elevating the head of the bed. DIF: Cognitive Level: Apply (application) REF: 744 TOP: Nursing Process: Evaluation MSC: NCLEX: Physiological Integrity

18. A patient with chronic heart failure who is taking a diuretic and an angiotensin-converting enzyme (ACE) inhibitor and who is on a low-sodium diet tells the home health nurse about a 5-lb weight gain in the past 3 days. The nurse's priority action will be to a. have the patient recall the dietary intake for the past 3 days. b. ask the patient about the use of the prescribed medications. c. assess the patient for clinical manifestations of acute heart failure. d. teach the patient about the importance of restricting dietary sodium.

ANS C The 5-lb weight gain over 3 days indicates that the patient's chronic heart failure may be worsening. It is important that the patient be assessed immediately for other clinical manifestations of decompensation, such as lung crackles. A dietary recall to detect hidden sodium in the diet, reinforcement of sodium restrictions, and assessment of medication compliance may be appropriate interventions but are not the first nursing actions indicated. DIF: Cognitive Level: Analyze (analysis) REF: 749 OBJ: Special Questions: Prioritization TOP: Nursing Process: Assessment MSC: NCLEX: Physiological Integrity

Which action by a new nurse who is giving fondaparinux (Arixtra) to a patient with a lower leg venous thromboembolism (VTE) indicates that more education about the drug is needed? a. The nurse avoids rubbing the injection site after giving the drug. b. The nurse injects the drug into the abdominal subcutaneous tissue. c. The nurse ejects the air bubble from the syringe before giving the drug. d. The nurse does not check partial thromboplastin time (PTT) before giving the drug.

ANS C The air bubble is not ejected before giving fondaparinux to avoid loss of drug. The other actions by the nurse are appropriate for subcutaneous administration of a low molecular weight heparin (LMWH). LMWHs typically do not require ongoing PTT monitoring and dose adjustment. DIF: Cognitive Level: Apply (application) REF: 820 TOP: Nursing Process: Implementation MSC: NCLEX: Safe and Effective Care Environment

The nurse is caring for a patient with a descending aortic dissection. Which assessment finding is most important to report to the health care provider? a. Weak pedal pulses b. Absent bowel sounds c. Blood pressure of 138/88 mm Hg d. 25 mL of urine output over the past hour

ANS C The blood pressure is typically kept at less than 120 mm Hg systolic to minimize extension of the dissection. The nurse will need to notify the health care provider so that -blockers or other antihypertensive drugs can be prescribed. The other findings are typical with aortic dissection and should also be reported but do not require immediate action. DIF: Cognitive Level: Analyze (analysis) REF: 815 OBJ: Special Questions: Prioritization TOP: Nursing Process: Assessment MSC: NCLEX: Physiological Integrity

1. While assessing a 68-yr-old with ascites, the nurse also notes jugular venous distention (JVD) with the head of the patient's bed elevated 45 degrees. The nurse knows this finding indicates a. decreased fluid volume. b. jugular vein atherosclerosis. c. increased right atrial pressure. d. incompetent jugular vein valves.

ANS C The jugular veins empty into the superior vena cava and then into the right atrium, so JVD with the patient sitting at a 45-degree angle reflects increased right atrial pressure. JVD is an indicator of excessive fluid volume (increased preload), not decreased fluid volume. JVD is not caused by incompetent jugular vein valves or atherosclerosis. DIF: Cognitive Level: Understand (comprehension) REF: 739 TOP: Nursing Process: Assessment MSC: NCLEX: Physiological Integrity

17. A patient with a history of chronic heart failure is admitted to the emergency department with severe dyspnea and a dry, hacking cough. Which action should the nurse do first? a. Auscultate the abdomen. b. Check the capillary refill. c. Auscultate the breath sounds. d. Ask about the patient's allergies.

ANS C This patient's severe dyspnea and cough indicate that acute decompensated heart failure (ADHF) is occurring. ADHF usually manifests as pulmonary edema, which should be detected and treated immediately to prevent ongoing hypoxemia and cardiac/respiratory arrest. The other assessments will provide useful data about the patient's volume status and also should be accomplished rapidly, but detection (and treatment) of pulmonary complications is the priority. DIF: Cognitive Level: Analyze (analysis) REF: 742 OBJ: Special Questions: Prioritization TOP: Nursing Process: Assessment MSC: NCLEX: Physiological Integrity

11. Following an acute myocardial infarction, a previously healthy 63-yr-old develops clinical manifestations of heart failure. The nurse anticipates discharge teaching will include information about a. Adrenergic blockers. b. calcium channel blockers. c. digitalis and potassium therapy regimens. d. angiotensin-converting enzyme (ACE) inhibitors.

ANS D ACE inhibitor therapy is currently recommended to prevent the development of heart failure in patients who have had a myocardial infarction and as a first-line therapy for patients with chronic heart failure. Digoxin therapy for heart failure is no longer considered a first-line measure, and digoxin is added to the treatment protocol when therapy with other drugs such as ACE-inhibitors, diuretics, and -adrenergic blockers is insufficient. Calcium channel blockers are not generally used in the treatment of heart failure. The -adrenergic blockers are not used as initial therapy for new onset heart failure. DIF: Cognitive Level: Apply (application) REF: 745 TOP: Nursing Process: Planning MSC: NCLEX: Physiological Integrity

When discussing risk factor modification for a patient who has a 5-cm abdominal aortic aneurysm, the nurse will focus teaching on which patient risk factor? a. Male gender c. Abdominal trauma history b. Turner syndrome d. Uncontrolled hypertension

ANS D All of the factors contribute to the patient's risk, but only hypertension can potentially be modified to decrease the patient's risk for further expansion of the aneurysm. DIF: Cognitive Level: Apply (application) REF: 810 TOP: Nursing Process: Implementation MSC: NCLEX: Physiological Integrity

16. A patient who has just been admitted with pulmonary edema is scheduled to receive the following medications. Which medication should the nurse question before giving? a. captopril 25 mg b. furosemide (Lasix) 60 mg c. digoxin (Lanoxin) 0.125 mg d. carvedilol (Coreg) 3.125 mg

ANS D Although carvedilol is appropriate for the treatment of chronic heart failure, it is not used for patients with acute decompensated heart failure (ADHF) because of the risk of worsening the heart failure. The other drugs are appropriate for the patient with ADHF. DIF: Cognitive Level: Analyze (analysis) REF: 748 TOP: Nursing Process: Implementation MSC: NCLEX: Physiological Integrity

Which nursing intervention for a patient who had an open repair of an abdominal aortic aneurysm 2 days previously is appropriate for the nurse to delegate to unlicensed assistive personnel (UAP)? a. Monitor the quality and presence of the pedal pulses. b. Teach the patient the signs of possible wound infection. c. Check the lower extremities for strength and movement. d. Help the patient to use a pillow to splint while coughing.

ANS D Assisting a patient who has already been taught how to cough is part of routine postoperative care and within the education and scope of practice for UAP. Patient teaching and assessment of essential postoperative functions such as circulation and movement should be done by RNs. DIF: Cognitive Level: Apply (application) REF: 824 OBJ: Special Questions: Delegation TOP: Nursing Process: Planning MSC: NCLEX: Safe and Effective Care Environment

2. The nurse is caring for a patient who is receiving IV furosemide (Lasix) and morphine for the treatment of acute decompensated heart failure (ADHF) with severe orthopnea. Which clinical finding is the best indicator that the treatment has been effective? a. Weight loss of 2 lb in 24 hours b. Hourly urine output greater than 60 mL c. Reduction in patient complaints of chest pain d. Reduced dyspnea with the head of bed at 30 degrees

ANS D Because the patient's major clinical manifestation of ADHF is orthopnea (caused by the presence of fluid in the alveoli), the best indicator that the medications are effective is a decrease in dyspnea with the head of the bed at 30 degrees. The other assessment data may also indicate that diuresis or improvement in cardiac output has occurred but are not as specific to evaluating this patient's response. DIF: Cognitive Level: Analyze (analysis) REF: 742 TOP: Nursing Process: Evaluation MSC: NCLEX: Physiological Integrity

A patient is being evaluated for postthrombotic syndrome. Which assessment will the nurse perform? a. Ask about leg pain with exercise. b. Determine the ankle-brachial index. c. Assess capillary refill in the patient's toes. d. Inspect for presence of lipodermatosclerosis.

ANS D Clinical signs of postthrombotic syndrome include lipodermatosclerosis. In this situation, the skin on the lower leg becomes scarred, and the leg becomes tapered like an "inverted bottle." The other assessments would be done for patients with peripheral arterial disease. DIF: Cognitive Level: Apply (application) REF: 818 TOP: Nursing Process: Evaluation MSC: NCLEX: Physiological Integrity

Which topic should the nurse include in patient teaching for a patient with a venous stasis ulcer on the left lower leg? a. Need to increase carbohydrate intake b. Methods of keeping the wound area dry c. Purpose of prophylactic antibiotic therapy d. Application of elastic compression stockings

ANS D Compression of the leg is essential to healing of venous stasis ulcers. High dietary intake of protein, rather than carbohydrates, is needed. Prophylactic antibiotics are not routinely used for venous ulcers. Moist dressings are used to hasten wound healing. DIF: Cognitive Level: Apply (application) REF: 826 TOP: Nursing Process: Planning MSC: NCLEX: Physiological Integrity

3. Which topic will the nurse plan to include in discharge teaching for a patient with heart failure with reduced ejection fraction (HFrEF)? a. Need to begin an aerobic exercise program several times weekly b. Use of salt substitutes to replace table salt when cooking and at the table c. Importance of making an annual appointment with the health care provider d. Benefits and side effects of angiotensin-converting enzyme (ACE) inhibitors

ANS D The core measures for the treatment of heart failure established by The Joint Commission indicate that patients with an ejection fraction below 40% should receive an ACE inhibitor to decrease the progression of heart failure. Aerobic exercise may not be appropriate for a patient with this level of heart failure, salt substitutes are not usually recommended because of the risk of hyperkalemia, and the patient will need to see the primary care provider more frequently than annually. DIF: Cognitive Level: Apply (application) REF: 737 TOP: Nursing Process: Planning MSC: NCLEX: Physiological Integrity

10. While admitting an 82-yr-old patient with acute decompensated heart failure to the hospital, the nurse learns that the patient lives alone and sometimes confuses the "water pill" with the "heart pill." When planning for the patient's discharge the nurse will facilitate a a. plan for around-the-clock care. b. consultation with a psychologist. c. transfer to a long-term care facility. d. referral to a home health care agency.

ANS D The data about the patient suggest that assistance in developing a system for taking medications correctly at home is needed. A home health nurse will assess the patient's home situation and help the patient develop a method for taking the two medications as directed. There is no evidence that the patient requires services such as a psychologist consult, long-term care, or around-the-clock home care. DIF: Cognitive Level: Apply (application) REF: 752 TOP: Nursing Process: Planning MSC: NCLEX: Physiological Integrity

Several hours after a patient had an open surgical repair of an abdominal aortic aneurysm, the UAP reports to the nurse that urinary output for the past 2 hours has been 45 mL. The nurse notifies the health care provider and anticipates an order for a(n) a. hemoglobin count. c. serum creatinine level. b. additional antibiotic. d. increased IV infusion rate.

ANS D The decreased urine output suggests decreased renal perfusion and monitoring of renal function is needed. There is no indication that infection is a concern, so antibiotic therapy and a WBC count are not needed. The IV rate may be increased because hypovolemia may be contributing to the patient's decreased urinary output. DIF: Cognitive Level: Apply (application) REF: 811 TOP: Nursing Process: Planning MSC: NCLEX: Physiological Integrity

A patient at the clinic says, "I always walk after dinner, but lately my leg cramps and hurts after just a few minutes of starting. The pain goes away after I stop walking, though." The nurse should a. look for the presence of tortuous veins bilaterally on the legs. b. ask about any skin color changes that occur in response to cold. c. assess for unilateral swelling, redness, and tenderness of either leg. d. palpate for the presence of dorsalis pedis and posterior tibial pulses.

ANS D The nurse should assess for other clinical manifestations of peripheral arterial disease in a patient who describes intermittent claudication. Changes in skin color that occur in response to cold are consistent with Raynaud's phenomenon. Tortuous veins on the legs suggest venous insufficiency. Unilateral leg swelling, redness, and tenderness indicate venous thromboembolism. DIF: Cognitive Level: Apply (application) REF: 814 TOP: Nursing Process: Assessment MSC: NCLEX: Physiological Integrity

23. An outpatient who has chronic heart failure returns to the clinic after 2 weeks of therapy with metoprolol (Toprol XL). Which assessment finding is most important for the nurse to report to the health care provider? a. 2+ bilateral pedal edema b. Heart rate of 56 beats/min c. Complaints of increased fatigue d. Blood pressure (BP) of 88/42 mm Hg

ANS D The patient's BP indicates that the dose of metoprolol may need to be decreased because of hypotension. Bradycardia is a frequent adverse effect of -adrenergic blockade, but the rate of 56 is not unusual though it may need to be monitored. -Adrenergic blockade initially will worsen symptoms of heart failure in many patients and patients should be taught that some increase in symptoms, such as fatigue and edema, is expected during the initiation of therapy with this class of drugs. DIF: Cognitive Level: Analyze (analysis) REF: 745 OBJ: Special Questions: Prioritization TOP: Nursing Process: Assessment MSC: NCLEX: Physiological Integrity

The health care provider has prescribed bed rest with the feet elevated for a patient admitted to the hospital with venous thromboembolism. Which action by the nurse to elevate the patient's feet is best? a. The patient is placed in the Trendelenburg position. b. Two pillows are positioned under the affected leg. c. The bed is elevated at the knee and pillows are placed under the feet. d. One pillow is placed under the thighs and two pillows are placed under the lower legs.

ANS D The purpose of elevating the feet is to enhance venous flow from the feet to the right atrium, which is best accomplished by placing two pillows under the feet and one under the thighs. Placing the patient in the Trendelenburg position will lower the head below heart level, which is not indicated for this patient. Placing pillows under the calf or elevating the bed at the knee may cause blood stasis at the calf level. DIF: Cognitive Level: Analyze (analysis) REF: 819 TOP: Nursing Process: Implementation MSC: NCLEX: Physiological Integrity

When preparing to defibrillate a patient. In which order will the nurse perform the following steps? (Put a comma and a space between each answer choice [A, B, C, D, E].) a. Turn the defibrillator on. b. Deliver the electrical charge. c. Select the appropriate energy level. d. Place the paddles on the patient's chest. e. Check the location of other staff and call out "all clear."

ANS: A, C, D, E, B This order will result in rapid defibrillation without endangering hospital staff.

Which action will the nurse include in the plan of care for a patient who was admitted with syncopal episodes of unknown origin? a. Instruct the patient to call for assistance before getting out of bed. b. Explain the association between various dysrhythmias and syncope. c. Educate the patient about the need to avoid caffeine and other stimulants. d. Tell the patient about the benefits of implantable cardioverter-defibrillators.

ANS: A A patient with fainting episodes is at risk for falls. The nurse will plan to minimize the risk by having assistance whenever the patient up. The other actions may be needed if dysrhythmias are found to be the cause of the patient's syncope, but are not appropriate for syncope of unknown origin

Which action by a new registered nurse (RN) who is orienting to the progressive care unit indicates a good understanding of the treatment of cardiac dysrhythmias? a. Injects IV adenosine (Adenocard) over 2 seconds to a patient with supraventricular tachycardia b. Obtains the defibrillator and quickly brings it to the bedside of a patient whose monitor shows asystole c. Turns the synchronizer switch to the "on" position before defibrillating a patient with ventricular fibrillation d. Gives the prescribed dose of diltiazem (Cardizem) to a patient with new-onset type II second degree AV block

ANS: A Adenosine must be given over 1 to 2 seconds to be effective. The other actions indicate a need for more education about treatment of cardiac dysrhythmias. The RN should hold the diltiazem until talking to the health care provider. The treatment for asystole is immediate CPR. The synchronizer switch should be "off" when defibrillating

29. When caring for a patient who has just arrived on the medical-surgical unit after having cardiac catheterization, which nursing intervention should the nurse delegate to a licensed practical/vocational nurse (LPN/LVN)? a. Give the scheduled aspirin and lipid-lowering medication. b. Perform the initial assessment of the catheter insertion site. c. Teach the patient about the usual postprocedure plan of care. d. Titrate the heparin infusion according to the agency protocol.

ANS: A Administration of oral medications is within the scope of practice for LPNs/LVNs. The initial assessment of the patient, patient teaching, and titration of IV anticoagulant medications should be done by the registered nurse (RN).

A patient with dilated cardiomyopathy has new onset atrial fibrillation that has been unresponsive to drug therapy for several days. The priority teaching needed for this patient would include information about a. anticoagulant therapy. b. permanent pacemakers. c. electrical cardioversion. d. IV adenosine (Adenocard).

ANS: A Atrial fibrillation therapy that has persisted for more than 48 hours requires anticoagulant treatment for 3 weeks before attempting cardioversion. This is done to prevent embolization of clots from the atria. Cardioversion may be done after several weeks of anticoagulation therapy. Adenosine is not used to treat atrial fibrillation. Pacemakers are routinely used for patients with bradydysrhythmias. Information does not indicate that the patient has a slow heart rate

After the nurse gives IV atropine to a patient with symptomatic type 1, second-degree atrioventricular (AV) block, which finding indicates that the medication has been effective? a. Increase in the patient's heart rate b. Increase in strength of peripheral pulses c. Decrease in premature atrial contractions d. Decrease in premature ventricular contractions

ANS: A Atropine will increase the heart rate and conduction through the AV node. Because the medication increases electrical conduction, not cardiac contractility, the quality of the peripheral pulses is not used to evaluate the drug effectiveness. The patient does not have premature atrial or ventricular contractions

26. The nurse is caring for a patient with aortic stenosis. Which assessment data obtained by the nurse would be most important to report to the health care provider? a. The patient complains of chest pressure when ambulating. b. A loud systolic murmur is heard along the right sternal border. c. A thrill is palpated at the second intercostal space, right sternal border. d. The point of maximum impulse (PMI) is at the left midclavicular line.

ANS: A Chest pressure (or pain) occurring with aortic stenosis is caused by cardiac ischemia, and reporting this information would be a priority. A systolic murmur and thrill are expected in a patient with aortic stenosis. A PMI at the left midclavicular line is normal. DIF: Cognitive Level: Analyze (analysis) REF: 793 OBJ: Special Questions: Prioritization TOP: Nursing Process: Assessment MSC: NCLEX: Physiological Integrity

34. Which information about a patient who has been receiving thrombolytic therapy for an acute myocardial infarction (AMI) is most important for the nurse to communicate to the health care provider? a. No change in the patient's chest pain b. An increase in troponin levels from baseline c. A large bruise at the patient's IV insertion site d. A decrease in ST-segment elevation on the electrocardiogram

ANS: A Continued chest pain suggests that the thrombolytic therapy is not effective and that other interventions such as percutaneous coronary intervention (PCI) may be needed. Bruising is a possible side effect of thrombolytic therapy, but it is not an indication that therapy should be discontinued. The decrease of the ST-segment elevation indicates that thrombolysis is occurring and perfusion is returning to the injured myocardium. An increase in troponin levels is expected with reperfusion and is related to the washout of cardiac markers into the circulation as the blocked vessel is opened.

A 20-year-old has a mandatory electrocardiogram (ECG) before participating on a college soccer team and is found to have sinus bradycardia, rate 52. Blood pressure (BP) is 114/54, and the student denies any health problems. What action by the nurse is most appropriate? a. Allow the student to participate on the soccer team. b. Refer the student to a cardiologist for further diagnostic testing. c. Tell the student to stop playing immediately if any dyspnea occurs. d. Obtain more detailed information about the student's family health history.

ANS: A In an aerobically trained individual, sinus bradycardia is normal. The student's normal BP and negative health history indicate that there is no need for a cardiology referral or for more detailed information about the family's health history. Dyspnea during an aerobic activity such as soccer is normal

A patient in the intensive care unit with acute decompensated heart failure (ADHF) complains of severe dyspnea and is anxious, tachypneic, and tachycardic. All of the following medications have been ordered for the patient. The nurse's priority action will be to a. give IV morphine sulfate 4 mg. b. give IV diazepam (Valium) 2.5 mg. c. increase nitroglycerin (Tridil) infusion by 5 mcg/min. d. increase dopamine (Intropin) infusion by 2 mcg/kg/min.

ANS: A Morphine improves alveolar gas exchange, improves cardiac output by reducing ventricular preload and afterload, decreases anxiety, and assists in reducing the subjective feeling of dyspnea. Diazepam may decrease patient anxiety, but it will not improve the cardiac output or gas exchange. Increasing the dopamine may improve cardiac output, but it will also increase the heart rate and myocardial oxygen consumption. Nitroglycerin will improve cardiac output and may be appropriate for this patient, but it will not directly reduce anxiety and will not act as quickly as morphine to decrease dyspnea

26. A patient who has recently started taking pravastatin (Pravachol) and niacin (Nicobid) reports the following symptoms to the nurse. Which is most important to communicate to the health care provider? a. Generalized muscle aches and pains b. Dizziness when changing positions quickly c. Nausea when taking the drugs before eating d. Flushing and pruritus after taking the medications

ANS: A Muscle aches and pains may indicate myopathy and rhabdomyolysis, which have caused acute kidney injury and death in some patients who have taken the statin medications. These symptoms indicate that the pravastatin may need to be discontinued. The other symptoms are common side effects when taking niacin, and although the nurse should follow-up with the health care provider, they do not indicate that a change in medication is needed.

Which action should the nurse include in the plan of care when caring for a patient admitted with acute decompensated heart failure (ADHF) who is receiving nesiritide (Natrecor)? a. Monitor blood pressure frequently. b. Encourage patient to ambulate in room. c. Titrate nesiritide slowly before stopping. d. Teach patient about home use of the drug.

ANS: A Nesiritide is a potent arterial and venous dilator, and the major adverse effect is hypotension. Because the patient is likely to have orthostatic hypotension, the patient should not be encouraged to ambulate. Nesiritide does not require titration and is used for ADHF but not in a home setting

16. While caring for a patient with aortic stenosis, the nurse identifies a nursing diagnosis of acute pain related to decreased coronary blood flow. An appropriate nursing intervention for this patient would be to a. promote rest to decrease myocardial oxygen demand. b. teach the patient about the need for anticoagulant therapy. c. teach the patient to use sublingual nitroglycerin for chest pain. d. raise the head of the bed 60 degrees to decrease venous return.

ANS: A Rest is recommended to balance myocardial oxygen supply and demand and to decrease chest pain. The patient with aortic stenosis requires higher preload to maintain cardiac output, so nitroglycerin and measures to decrease venous return are contraindicated. Anticoagulation is not recommended unless the patient has atrial fibrillation. DIF: Cognitive Level: Apply (application) REF: 796 TOP: Nursing Process: Implementation MSC: NCLEX: Physiological Integrity

27. A patient who is being admitted to the emergency department with intermittent chest pain gives the following list of medications to the nurse. Which medication has the most immediate implications for the patient's care? a. Sildenafil (Viagra) b. Furosemide (Lasix) c. Captopril (Capoten) d. Warfarin (Coumadin)

ANS: A The nurse will need to avoid giving nitrates to the patient because nitrate administration is contraindicated in patients who are using sildenafil because of the risk of severe hypotension caused by vasodilation. The other home medications also should be documented and reported to the health care provider but do not have as immediate an impact on decisions about the patient's treatment.

19. Three days after experiencing a myocardial infarction (MI), a patient who is scheduled for discharge asks for assistance with hygiene activities, saying, "I am too nervous to take care of myself." Based on this information, which nursing diagnosis is appropriate? a. Ineffective coping related to anxiety b. Activity intolerance related to weakness c. Denial related to lack of acceptance of the MI d. Disturbed personal identity related to understanding of illness

ANS: A The patient data indicate that ineffective coping after the MI caused by anxiety about the impact of the MI is a concern. The other nursing diagnoses may be appropriate for some patients after an MI, but the data for this patient do not support denial, activity intolerance, or disturbed personal identity.

Which intervention by a new nurse who is caring for a patient who has just had an implantable cardioverter-defibrillator (ICD) inserted indicates a need for more education about care of patients with ICDs? a. The nurse assists the patient to do active range of motion exercises for all extremities. b. The nurse assists the patient to fill out the application for obtaining a Medic Alert ID. c. The nurse gives amiodarone (Cordarone) to the patient without first consulting with the health care provider. d. The nurse teaches the patient that sexual activity usually can be resumed once the surgical incision is healed.

ANS: A The patient should avoid moving the arm on the ICD insertion site until healing has occurred in order to prevent displacement of the ICD leads. The other actions by the new nurse are appropriate for this patient.

After receiving change-of-shift report on a heart failure unit, which patient should the nurse assess first? a. A patient who is cool and clammy, with new-onset confusion and restlessness b. A patient who has crackles bilaterally in the lung bases and is receiving oxygen. c. A patient who had dizziness after receiving the first dose of captopril (Capoten) d. A patient who is receiving IV nesiritide (Natrecor) and has a blood pressure of 100/62

ANS: A The patient who has "wet-cold" clinical manifestations of heart failure is perfusing inadequately and needs rapid assessment and changes in management. The other patients also should be assessed as quickly as possible but do not have indications of severe decreases in tissue perfusion.

27. Two days after an acute myocardial infarction (MI), a patient complains of stabbing chest pain that increases with a deep breath. Which action will the nurse take first? a. Auscultate the heart sounds. b. Check the patient's temperature. c. Give the PRN acetaminophen (Tylenol). d. Notify the patient's health care provider.

ANS: A The patient's clinical manifestations and history are consistent with pericarditis, and the first action by the nurse should be to listen for a pericardial friction rub. Checking the temperature and notifying the health care provider are also appropriate actions but would not be done before listening for a rub. Acetaminophen (Tylenol) is not very effective for pericarditis pain, and an analgesic would not be given before assessment of a new symptom. DIF: Cognitive Level: Analyze (analysis) REF: 785 OBJ: Special Questions: Prioritization TOP: Nursing Process: Implementation MSC: NCLEX: Physiological Integrity

10. A patient with rheumatic fever has subcutaneous nodules, erythema marginatum, and polyarthritis. The patient reports that discomfort in the joints prevents favorite activities such as taking a daily walk and working on sewing projects. Based on these findings, which nursing diagnosis statement would be appropriate? a. Activity intolerance related to arthralgia b. Anxiety related to permanent joint fixation c. Altered body image related to polyarthritis d. Social isolation related to pain and swelling

ANS: A The patient's joint pain will lead to difficulty with activity. Although acute joint pain will be a problem for this patient, joint inflammation is a temporary clinical manifestation of rheumatic fever and is not associated with permanent joint changes. This patient did not provide any data to support a diagnosis of social isolation, anxiety, or altered body image. DIF: Cognitive Level: Apply (application) REF: 790 TOP: Nursing Process: Diagnosis MSC: NCLEX: Physiological Integrity

During a visit to a 78-year-old with chronic heart failure, the home care nurse finds that the patient has ankle edema, a 2-kg weight gain over the past 2 days, and complains of "feeling too tired to get out of bed." Based on these data, the best nursing diagnosis for the patient is a. activity intolerance related to fatigue. b. disturbed body image related to weight gain. c. impaired skin integrity related to ankle edema. d. impaired gas exchange related to dyspnea on exertion.

ANS: A The patient's statement supports the diagnosis of activity intolerance. There are no data to support the other diagnoses, although the nurse will need to assess for other patient problems

4. When planning care for a patient hospitalized with a streptococcal infective endocarditis (IE), which intervention is most appropriate for the nurse to include? a. Arrange for placement of a long-term IV catheter. b. Monitor labs for levels of streptococcal antibodies. c. Teach the importance of completing all oral antibiotics. d. Encourage the patient to begin regular aerobic exercise.

ANS: A Treatment for IE involves 4 to 6 weeks of IV antibiotic therapy to eradicate the bacteria, which will require a long-term IV catheter such as a peripherally inserted central catheter (PICC) line. Rest periods and limiting physical activity to a moderate level are recommended during the treatment for IE. Oral antibiotics are not effective in eradicating the infective bacteria that cause IE. Blood cultures, rather than antibody levels, are used to monitor the effectiveness of antibiotic therapy. DIF: Cognitive Level: Apply (application) REF: 784 TOP: Nursing Process: Planning MSC: NCLEX: Physiological Integrity

21. A few days after experiencing a myocardial infarction (MI) and successful percutaneous coronary intervention, the patient states, "I just had a little chest pain. As soon as I get out of here, I'm going for my vacation as planned." Which reply would be most appropriate for the nurse to make? a. "What do you think caused your chest pain?" b. "Where are you planning to go for your vacation?" c. "Sometimes plans need to change after a heart attack." d. "Recovery from a heart attack takes at least a few weeks."

ANS: A When the patient is experiencing denial, the nurse should assist the patient in testing reality until the patient has progressed beyond this step of the emotional adjustment to MI. Asking the patient about vacation plans reinforces the patient's plan, which is not appropriate in the immediate post-MI period. Reminding the patient in denial about the MI is likely to make the patient angry and lead to distrust of the nursing staff.

Based on the Joint Commission Core Measures for patients with heart failure, which topics should the nurse include in the discharge teaching plan for a patient who has been hospitalized with chronic heart failure (select all that apply)? a. How to take and record daily weight b. Importance of limiting aerobic exercise c. Date and time of follow-up appointment d. Symptoms indicating worsening heart failure e. Actions and side effects of prescribed medications

ANS: A, C, D, E The Joint Commission Core Measures state that patients should be taught about prescribed medications, follow-up appointments, weight monitoring, and actions to take for worsening symptoms. Patients with heart failure are encouraged to begin or continue aerobic exercises such as walking, while self-monitoring to avoid excessive fatigue

33. When admitting a patient with a non-ST-segment-elevation myocardial infarction (NSTEMI) to the intensive care unit, which action should the nurse perform first? a. Obtain the blood pressure. b. Attach the cardiac monitor. c. Assess the peripheral pulses. d. Auscultate the breath sounds.

ANS: B Because dysrhythmias are the most common complication of myocardial infarction (MI), the first action should be to place the patient on a cardiac monitor. The other actions also are important and should be accomplished as quickly as possible.

3. Which assessment data collected by the nurse who is admitting a patient with chest pain suggest that the pain is caused by an acute myocardial infarction (AMI)? a. The pain increases with deep breathing. b. The pain has lasted longer than 30 minutes. c. The pain is relieved after the patient takes nitroglycerin. d. The pain is reproducible when the patient raises the arms.

ANS: B Chest pain that lasts for 20 minutes or more is characteristic of AMI. Changes in pain that occur with raising the arms or with deep breathing are more typical of musculoskeletal pain or pericarditis. Stable angina is usually relieved when the patient takes nitroglycerin.

28. The nurse is caring for a 64-yr-old patient admitted with mitral valve regurgitation. Which information obtained by the nurse when assessing the patient should be communicated to the health care provider immediately? a. The patient has 4+ peripheral edema. b. The patient has diffuse bilateral crackles. c. The patient has a loud systolic murmur across the precordium. d. The patient has a palpable thrill felt over the left anterior chest.

ANS: B Crackles that are audible throughout the lungs indicate that the patient is experiencing severe left ventricular failure with pulmonary congestion and needs immediate interventions such as diuretics. A systolic murmur and palpable thrill would be expected in a patient with mitral regurgitation. Although 4+ peripheral edema indicates a need for a change in therapy, it does not need to be addressed urgently. DIF: Cognitive Level: Analyze (analysis) REF: 795 OBJ: Special Questions: Prioritization TOP: Nursing Process: Assessment MSC: NCLEX: Physiological Integrity

3. The nurse identifies the nursing diagnosis of decreased cardiac output related to valvular insufficiency for the patient with infective endocarditis (IE) based on which assessment finding(s)? a. Fever, chills, and diaphoresis b. Urine output less than 30 mL/hr c. Petechiae on the inside of the mouth and conjunctiva d. Increase in heart rate of 15 beats/minute with walking

ANS: B Decreased renal perfusion caused by inadequate cardiac output will lead to decreased urine output. Petechiae, fever, chills, and diaphoresis are symptoms of IE but are not caused by decreased cardiac output. An increase in pulse rate of 15 beats/min is normal with exercise. DIF: Cognitive Level: Apply (application) REF: 780 TOP: Nursing Process: Diagnosis MSC: NCLEX: Physiological Integrity

20. When caring for a patient who is recovering from a sudden cardiac death (SCD) event and has no evidence of an acute myocardial infarction (AMI), the nurse will anticipate teaching the patient that a. sudden cardiac death events rarely reoccur. b. additional diagnostic testing will be required. c. long-term anticoagulation therapy will be needed. d. limited physical activity after discharge will be needed to prevent future events.

ANS: B Diagnostic testing (e.g., stress test, Holter monitor, electrophysiologic studies, cardiac catheterization) is used to determine the possible cause of the SCD and treatment options. SCD is likely to recur. Anticoagulation therapy will not have any effect on the incidence of SCD, and SCD can occur even when the patient is resting.

5. A patient is admitted to the hospital with possible acute pericarditis. The nurse should plan to teach the patient about the purpose of a. blood cultures. c. cardiac catheterization. b. echocardiography. d. 24-hour Holter monitor.

ANS: B Echocardiograms are useful in detecting the presence of the pericardial effusions associated with pericarditis. Blood cultures are not indicated unless the patient has evidence of sepsis. Cardiac catheterization and 24-hour Holter monitor are not diagnostic procedures for pericarditis. DIF: Cognitive Level: Apply (application) REF: 786 TOP: Nursing Process: Planning MSC: NCLEX: Physiological Integrity

32. After receiving change-of-shift report on four patients, which patient should the nurse assess first? a. Patient with rheumatic fever who has sharp chest pain with a deep breath b. Patient with acute aortic regurgitation whose blood pressure is 86/54 mm Hg c. Patient with infective endocarditis who has a murmur and splinter hemorrhages d. Patient with dilated cardiomyopathy who has bilateral crackles at the lung bases

ANS: B Hypotension in patients with acute aortic regurgitation may indicate cardiogenic shock. The nurse should immediately assess this patient for other findings such as dyspnea, chest pain or tachycardia. The findings in the other patients are typical of their diagnoses and do not indicate a need for urgent assessment and intervention. DIF: Cognitive Level: Analyze (analysis) REF: 792 OBJ: Special Questions: Multiple Patients | Special Questions: Prioritization TOP: Nursing Process: Assessment MSC: NCLEX: Safe and Effective Care Environment

A 19-year-old student comes to the student health center at the end of the semester complaining that, "My heart is skipping beats." An electrocardiogram (ECG) shows occasional premature ventricular contractions (PVCs). What action should the nurse take next? a. Start supplemental O2 at 2 to 3 L/min via nasal cannula. b. Ask the patient about current stress level and caffeine use. c. Ask the patient about any history of coronary artery disease. d. Have the patient taken to the hospital emergency department (ED).

ANS: B In a patient with a normal heart, occasional PVCs are a benign finding. The timing of the PVCs suggests stress or caffeine as possible etiologic factors. It is unlikely that the patient has coronary artery disease, and this should not be the first question the nurse asks. The patient is hemodynamically stable, so there is no indication that the patient needs to be seen in the ED or that oxygen needs to be administered

7. After the nurse teaches the patient about the use of carvedilol (Coreg) in preventing anginal episodes, which statement by a patient indicates that the teaching has been effective? a. "Carvedilol will help my heart muscle work harder." b. "It is important not to suddenly stop taking the carvedilol." c. "I can expect to feel short of breath when taking carvedilol." d. "Carvedilol will increase the blood flow to my heart muscle."

ANS: B Patients who have been taking β-adrenergic blockers can develop intense and frequent angina if the medication is suddenly discontinued. Carvedilol (Coreg) decreases myocardial contractility. Shortness of breath that occurs when taking β-adrenergic blockers for angina may be due to bronchospasm and should be reported to the health care provider. Carvedilol works by decreasing myocardial oxygen demand, not by increasing blood flow to the coronary arteries.

6. To assess the patient with pericarditis for evidence of a pericardial friction rub, the nurse should a. listen for a rumbling, low-pitched, systolic murmur over the left anterior chest. b. auscultate with the diaphragm of the stethoscope on the lower left sternal border. c. ask the patient to cough during auscultation to distinguish the sound from a pleural friction rub. d. feel the precordial area with the palm of the hand to detect vibrations with cardiac contraction.

ANS: B Pericardial friction rubs are best heard with the diaphragm at the lower left sternal border. The nurse should ask the patient to hold his or her breath during auscultation to distinguish the sounds from a pleural friction rub. Friction rubs are not typically low pitched or rumbling and are not confined to systole. Rubs are not assessed by palpation. DIF: Cognitive Level: Understand (comprehension) REF: 785 TOP: Nursing Process: Assessment MSC: NCLEX: Physiological Integrity

7. The nurse suspects cardiac tamponade in a patient who has acute pericarditis. To assess for the presence of pulsus paradoxus, the nurse should a. subtract the diastolic blood pressure from the systolic blood pressure. b. note when Korotkoff sounds are auscultated during both inspiration and expiration. c. check the electrocardiogram (ECG) for variations in rate during the respiratory cycle. d. listen for a pericardial friction rub that persists when the patient is instructed to stop breathing.

ANS: B Pulsus paradoxus exists when there is a gap of greater than 10 mm Hg between when Korotkoff sounds can be heard during only expiration and when they can be heard throughout the respiratory cycle. The other methods described would not be useful in determining the presence of pulsus paradoxus. DIF: Cognitive Level: Apply (application) REF: 785 TOP: Nursing Process: Assessment MSC: NCLEX: Physiological Integrity

23. A patient who is recovering from an acute myocardial infarction (AMI) asks the nurse about when sexual intercourse can be resumed. Which response by the nurse is best? a. "Most patients are able to enjoy intercourse without any complications." b. "Sexual activity uses about as much energy as climbing two flights of stairs." c. "The doctor will provide sexual guidelines when your heart is strong enough." d. "Holding and cuddling are good ways to maintain intimacy after a heart attack."

ANS: B Sexual activity places about as much physical stress on the cardiovascular system as most moderate-energy activities such as climbing two flights of stairs. The other responses do not directly address the patient's question or may not be accurate for this patient.

24. Which assessment finding in a patient who is admitted with infective endocarditis (IE) is most important to communicate to the health care provider? a. Generalized muscle aching c. Janeway's lesions on the palms b. Sudden onset right flank pain d. Temperature 100.7°F (38.1°C)

ANS: B Sudden onset of flank pain indicates possible embolization to the kidney and may require diagnostic testing such as a renal arteriogram and interventions to improve renal perfusion. The other findings are typically found in IE but do not require any new interventions. DIF: Cognitive Level: Analyze (analysis) REF: 781 OBJ: Special Questions: Prioritization TOP: Nursing Process: Assessment MSC: NCLEX: Physiological Integrity

28. Which assessment finding by the nurse caring for a patient who has had coronary artery bypass grafting using a right radial artery graft is most important to communicate to the health care provider? a. Complaints of incisional chest pain b. Pallor and weakness of the right hand c. Fine crackles heard at both lung bases d. Redness on both sides of the sternal incision

ANS: B The changes in the right hand indicate compromised blood flow, which requires immediate evaluation and actions such as prescribed calcium channel blockers or surgery. The other changes are expected and/or require nursing interventions.

13. When titrating IV nitroglycerin (Tridil) for a patient with a myocardial infarction (MI), which action will the nurse take to evaluate the effectiveness of the medication? a. Monitor heart rate. b. Ask about chest pain. c. Check blood pressure. d. Observe for dysrhythmias.

ANS: B The goal of IV nitroglycerin administration in MI is relief of chest pain by improving the balance between myocardial oxygen supply and demand. The nurse also will monitor heart rate and blood pressure (BP) and observe for dysrhythmias, but these parameters will not indicate whether the medication is effective.

40. Which patient at the cardiovascular clinic requires the most immediate action by the nurse? a. Patient with type 2 diabetes whose current blood glucose level is 145 mg/dL b. Patient with stable angina whose chest pain has recently increased in frequency c. Patient with familial hypercholesterolemia and a total cholesterol of 465 mg/dL d. Patient with chronic hypertension whose blood pressure today is 172/98 mm Hg

ANS: B The history of more frequent chest pain suggests that the patient may have unstable angina, which is part of the acute coronary syndrome spectrum. This will require rapid implementation of actions such as cardiac catheterization and possible percutaneous coronary intervention. The data about the other patients suggest that their conditions are stable.

12. When developing a community health program to decrease the incidence of rheumatic fever, which action should the community health nurse include? a. Vaccinate high-risk groups in the community with streptococcal vaccine. b. Teach community members to seek treatment for streptococcal pharyngitis. c. Teach about the importance of monitoring temperature when sore throats occur. d. Teach about prophylactic antibiotics to those with a family history of rheumatic fever.

ANS: B The incidence of rheumatic fever is decreased by treatment of streptococcal infections with antibiotics. Family history is not a risk factor for rheumatic fever. There is no immunization that is effective in decreasing the incidence of rheumatic fever. Teaching about monitoring temperature will not decrease the incidence of rheumatic fever. DIF: Cognitive Level: Apply (application) REF: 790 TOP: Nursing Process: Planning MSC: NCLEX: Health Promotion and Maintenance

33. After receiving information about four patients during change-of-shift report, which patient should the nurse assess first? a. Patient with acute pericarditis who has a pericardial friction rub b. Patient who has just returned to the unit after balloon valvuloplasty c. Patient who has hypertrophic cardiomyopathy and a heart rate of 116 d. Patient with a mitral valve replacement who has an anticoagulant scheduled

ANS: B The patient who has just arrived after balloon valvuloplasty will need assessment for complications such as bleeding and hypotension. The information about the other patients is consistent with their diagnoses and does not indicate any complications or need for urgent assessment or intervention. DIF: Cognitive Level: Analyze (analysis) REF: 793 OBJ: Special Questions: Prioritization | Special Questions: Multiple Patients TOP: Nursing Process: Planning MSC: NCLEX: Safe and Effective Care Environment

A patient whose heart monitor shows sinus tachycardia, rate 132, is apneic and has no palpable pulses. What is the first action that the nurse should take? a. Perform synchronized cardioversion. b. Start cardiopulmonary resuscitation (CPR). c. Administer atropine per agency dysrhythmia protocol. d. Provide supplemental oxygen via non-rebreather mask.

ANS: B The patient's clinical manifestations indicate pulseless electrical activity and the nurse should immediately start CPR. The other actions would not be of benefit to this patient

After receiving change-of-shift report on a heart failure unit, which patient should the nurse assess first? a. Patient who is taking carvedilol (Coreg) and has a heart rate of 58 b. Patient who is taking digoxin and has a potassium level of 3.1 mEq/L c. Patient who is taking isosorbide dinitrate/hydralazine (BiDil) and has a headache d. Patient who is taking captopril (Capoten) and has a frequent nonproductive cough

ANS: B The patient's low potassium level increases the risk for digoxin toxicity and potentially fatal dysrhythmias. The nurse should assess the patient for other signs of digoxin toxicity and then notify the health care provider about the potassium level. The other patients also have side effects of their medications, but their symptoms do not indicate potentially life-threatening complications

23. Which admission order written by the health care provider for a patient admitted with infective endocarditis (IE) and a fever would be a priority for the nurse to implement? a. Administer ceftriaxone 1 g IV. b. Order blood cultures drawn from two sites. c. Give acetaminophen (Tylenol) PRN for fever. d. Arrange for a transesophageal echocardiogram.

ANS: B Treatment of the IE with antibiotics should be started as quickly as possible, but it is essential to obtain blood cultures before starting antibiotic therapy to obtain accurate sensitivity results. The echocardiogram and acetaminophen administration also should be implemented rapidly, but the blood cultures (and then administration of the antibiotic) have the highest priority. DIF: Cognitive Level: Analyze (analysis) REF: 782 OBJ: Special Questions: Prioritization TOP: Nursing Process: Implementation MSC: NCLEX: Physiological Integrity

Which action should the nurse perform when preparing a patient with supraventricular tachycardia for cardioversion who is alert and has a blood pressure of 110/66 mm Hg? a. Turn the synchronizer switch to the "off" position. b. Give a sedative before cardioversion is implemented. c. Set the defibrillator/cardioverter energy to 360 joules. d. Provide assisted ventilations with a bag-valve-mask device.

ANS: B When a patient has a nonemergency cardioversion, sedation is used just before the procedure. The synchronizer switch is turned "on" for cardioversion. The initial level of joules for cardioversion is low (e.g., 50). Assisted ventilations are not indicated for this patient

22. When evaluating the effectiveness of preoperative teaching with a patient scheduled for coronary artery bypass graft (CABG) surgery using the internal mammary artery, the nurse determines that additional teaching is needed when the patient says which of the following? a. "They will circulate my blood with a machine during the surgery." b. "I will have small incisions in my leg where they will remove the vein." c. "They will use an artery near my heart to go around the area that is blocked." d. "I will need to take an aspirin every day after the surgery to keep the graft open."

ANS: B When the internal mammary artery is used there is no need to have a saphenous vein removed from the leg. The other statements by the patient are accurate and indicate that the teaching has been effective.

15. Following an acute myocardial infarction (AMI), a patient ambulates in the hospital hallway. When the nurse is evaluating the patient's response to the activity, which assessment data would indicate that the exercise level should be decreased? a. Blood pressure (BP) changes from 118/60 to 126/68 mm Hg. b. Oxygen saturation drops from 99% to 95%. c. Heart rate increases from 66 to 92 beats/minute. d. Respiratory rate goes from 14 to 20 breaths/minute.

ANS: C A change in heart rate of more than 20 beats over the resting heart rate indicates that the patient should stop and rest. The increases in BP and respiratory rate, and the slight decrease in oxygen saturation, are normal responses to exercise.

32. A patient admitted to the coronary care unit (CCU) with an ST-segment-elevation myocardial infarction (STEMI) is restless and anxious. The blood pressure is 86/40 and heart rate is 123. Based on this information, which nursing diagnosis is a priority for the patient? a. Acute pain related to myocardial infarction b. Anxiety related to perceived threat of death c. Stress overload related to acute change in health d. Decreased cardiac output related to cardiogenic shock

ANS: C All the nursing diagnoses may be appropriate for this patient, but the hypotension and tachycardia indicate decreased cardiac output and shock from the damaged myocardium. This will result in decreased perfusion to all vital organs (e.g., brain, kidney, heart) and is a priority.

A patient who is receiving dobutamine (Dobutrex) for the treatment of acute decompensated heart failure (ADHF) has the following nursing interventions included in the plan of care. Which action will be most appropriate for the registered nurse (RN) to delegate to an experienced licensed practical/vocational nurse (LPN/LVN)? a. Assess the IV insertion site for signs of extravasation. b. Teach the patient the reasons for remaining on bed rest. c. Monitor the patient's blood pressure and heart rate every hour. d. Titrate the rate to keep the systolic blood pressure >90 mm Hg.

ANS: C An experienced LPN/LVN would be able to monitor BP and heart rate and would know to report significant changes to the RN. Teaching patients, making adjustments to the drip rate for vasoactive medications, and monitoring for serious complications such as extravasation require RN level education and scope of practice

17. During discharge teaching with an older patient who had a mitral valve replacement with a mechanical valve, the nurse must instruct the patient on the a. use of daily aspirin for anticoagulation. b. correct method for taking the radial pulse. c. need for frequent laboratory blood testing. d. need to avoid any physical activity for 1 month.

ANS: C Anticoagulation with warfarin (Coumadin) is needed for a patient with mechanical valves to prevent clotting on the valve. This will require frequent international normalized ratio testing. Daily aspirin use will not be effective in reducing the risk for clots on the valve. Monitoring of the radial pulse is not necessary after valve replacement. Patients should resume activities of daily living as tolerated. DIF: Cognitive Level: Apply (application) REF: 796 TOP: Nursing Process: Implementation MSC: NCLEX: Physiological Integrity

The nurse is caring for a patient immediately after repair of an abdominal aortic aneurysm. On assessment, the patient has absent popliteal, posterior tibial, and dorsalis pedis pulses. The legs are cool and mottled. Which action should the nurse take first? a. Notify the surgeon and anesthesiologist. b. Wrap both the legs in a warming blanket. c. Document the findings and recheck in 15 minutes. d. Compare findings to the preoperative assessment of the pulses.

ANS A Lower extremity pulses may be absent for a short time after surgery because of vasospasm and hypothermia. Decreased or absent pulses together with a cool and mottled extremity may indicate embolization or graft occlusion. These findings should be reported to the surgeon immediately because this is an emergency situation. Because pulses are marked before surgery, the nurse would know whether pulses were present before surgery before notifying the health care providers about the absent pulses. Because the patient's symptoms may indicate graft occlusion or multiple emboli and a possible need to return to surgery, it is not appropriate to wait 15 minutes before taking action. A warming blanket will not improve the circulation to the patient's legs. DIF: Cognitive Level: Analyze (analysis) REF: 814 OBJ: Special Questions: Prioritization TOP: Nursing Process: Implementation MSC: NCLEX: Physiological Integrity

Which diagnostic test will be most useful to the nurse in determining whether a patient admitted with acute shortness of breath has heart failure? a. Serum troponin b. Arterial blood gases c. B-type natriuretic peptide d. 12-lead electrocardiogram

ANS: C B-type natriuretic peptide (BNP) is secreted when ventricular pressures increase, as they do with heart failure. Elevated BNP indicates a probable or very probable diagnosis of heart failure. A twelve-lead electrocardiogram, arterial blood gases, and troponin may also be used in determining the causes or effects of heart failure but are not as clearly diagnostic of heart failure as BNP

29. Which action by the nurse will determine if the therapies ordered for a patient with chronic constrictive pericarditis are most effective? a. Assess for the presence of a paradoxical pulse. b. Monitor for changes in the patient's sedimentation rate. c. Assess for the presence of jugular venous distention (JVD). d. Check the electrocardiogram (ECG) for ST segment changes.

ANS: C Because the most common finding on physical examination for a patient with chronic constrictive pericarditis is jugular venous distention, a decrease in JVD indicates improvement. Paradoxical pulse, ST segment ECG changes, and changes in sedimentation rates occur with acute pericarditis but are not expected in chronic constrictive pericarditis. DIF: Cognitive Level: Apply (application) REF: 787 TOP: Nursing Process: Evaluation MSC: NCLEX: Physiological Integrity

A patient who is on the progressive care unit develops atrial flutter, rate 150, with associated dyspnea and chest pain. Which action that is included in the hospital dysrhythmia protocol should the nurse do first? a. Obtain a 12-lead electrocardiogram (ECG). b. Notify the health care provider of the change in rhythm. c. Give supplemental O2 at 2 to 3 L/min via nasal cannula. d. Assess the patient's vital signs including oxygen saturation.

ANS: C Because this patient has dyspnea and chest pain in association with the new rhythm, the nurse's initial actions should be to address the patient's airway, breathing, and circulation (ABC) by starting with oxygen administration. The other actions also are important and should be implemented rapidly

A patient with heart failure has a new order for captopril (Capoten) 12.5 mg PO. After administering the first dose and teaching the patient about the drug, which statement by the patient indicates that teaching has been effective? a. "I will be sure to take the medication with food." b. "I will need to eat more potassium-rich foods in my diet." c. "I will call for help when I need to get up to use the bathroom." d. "I will expect to feel more short of breath for the next few days."

ANS: C Captopril can cause hypotension, especially after the initial dose, so it is important that the patient not get up out of bed without assistance until the nurse has had a chance to evaluate the effect of the first dose. The angiotensin-converting enzyme (ACE) inhibitors are potassium sparing, and the nurse should not teach the patient to purposely increase sources of dietary potassium. Increased shortness of breath is expected with the initiation of b-adrenergic blocker therapy for heart failure, not for ACE inhibitor therapy. ACE inhibitors are best absorbed when taken an hour before eating

12. Heparin is ordered for a patient with a non-ST-segment-elevation myocardial infarction (NSTEMI). What is the purpose of the heparin? a. Heparin enhances platelet aggregation. b. Heparin decreases coronary artery plaque size. c. Heparin prevents the development of new clots in the coronary arteries. d. Heparin dissolves clots that are blocking blood flow in the coronary arteries.

ANS: C Heparin helps prevent the conversion of fibrinogen to fibrin and decreases coronary artery thrombosis. It does not change coronary artery plaque, dissolve already formed clots, or enhance platelet aggregation.

A patient has recently started on digoxin (Lanoxin) in addition to furosemide (Lasix) and captopril (Capoten) for the management of heart failure. Which assessment finding by the home health nurse is a priority to communicate to the health care provider? a. Presence of 1 to 2+ edema in the feet and ankles b. Palpable liver edge 2 cm below the ribs on the right side c. Serum potassium level 3.0 mEq/L after 1 week of therapy d. Weight increase from 120 pounds to 122 pounds over 3 days

ANS: C Hypokalemia can predispose the patient to life-threatening dysrhythmias (e.g., premature ventricular contractions), and potentiate the actions of digoxin and increase the risk for digoxin toxicity, which can also cause life-threatening dysrhythmias. The other data indicate that the patient's heart failure requires more effective therapies, but they do not require nursing action as rapidly as the low serum potassium level

A patient has a junctional escape rhythm on the monitor. The nurse will expect the patient to have a heart rate of _____ beats/minute. a. 15 to 20 b. 20 to 40 c. 40 to 60 d. 60 to 100

ANS: C If the sinoatrial (SA) node fails to discharge, the atrioventricular (AV) node will automatically discharge at the normal rate of 40 to 60 beats/minute. The slower rates are typical of the bundle of His and the Purkinje system and may be seen with failure of both the SA and AV node to discharge. The normal SA node rate is 60 to 100 beats/minute

19. A patient in the intensive care unit with acute decompensated heart failure (ADHF) complains of severe dyspnea and is anxious, tachypneic, and tachycardic. Several drugs have been ordered for the patient. The nurse's priority action will be to a. give PRN IV morphine sulfate 4 mg. b. give PRN IV diazepam (Valium) 2.5 mg. c. increase nitroglycerin infusion by 5 mcg/min. d. increase dopamine infusion by 2 mcg/kg/min.

ANS A Morphine improves alveolar gas exchange, improves cardiac output by reducing ventricular preload and afterload, decreases anxiety, and assists in reducing the subjective feeling of dyspnea. Diazepam may decrease patient anxiety, but it will not improve the cardiac output or gas exchange. Increasing the dopamine may improve cardiac output, but it will also increase the heart rate and myocardial oxygen consumption. Nitroglycerin will improve cardiac output and may be appropriate for this patient, but it will not directly reduce anxiety and will not act as quickly as morphine to decrease dyspnea. DIF: Cognitive Level: Analyze (analysis) REF: 745 OBJ: Special Questions: Prioritization TOP: Nursing Process: Implementation MSC: NCLEX: Physiological Integrity

14. Which action should the nurse include in the plan of care when caring for a patient admitted with acute decompensated heart failure (ADHF) who is receiving nesiritide (Natrecor)? a. Monitor blood pressure frequently. b. Encourage patient to ambulate in room. c. Titrate nesiritide slowly before stopping. d. Teach patient about home use of the drug.

ANS A Nesiritide is a potent arterial and venous dilator, and the major adverse effect is hypotension. Because the patient is likely to have orthostatic hypotension, the patient should not be encouraged to ambulate. Nesiritide does not require titration and is used for ADHF but not in a home setting. DIF: Cognitive Level: Apply (application) REF: 744 TOP: Nursing Process: Planning MSC: NCLEX: Physiological Integrity

After teaching a patient with newly diagnosed Raynaud's phenomenon about how to manage the condition, which action by the patient best demonstrates that the teaching has been effective? a. The patient exercises indoors during the winter months. b. The patient immerses hands in hot water when they turn pale. c. The patient takes pseudoephedrine (Sudafed) for cold symptoms. d. The patient avoids taking nonsteroidal antiinflammatory drugs (NSAIDs).

ANS A Patients should avoid temperature extremes by exercising indoors when it is cold. To avoid burn injuries, the patient should use warm rather than hot water to warm the hands. Pseudoephedrine is a vasoconstrictor and should be avoided. There is no reason to avoid taking NSAIDs with Raynaud's phenomenon. DIF: Cognitive Level: Apply (application) REF: 809 TOP: Nursing Process: Evaluation MSC: NCLEX: Physiological Integrity

A patient who is 2 days post femoral popliteal bypass graft to the right leg is being cared for on the vascular unit. Which action by a licensed practical/vocational nurse (LPN/LVN) caring for the patient requires the registered nurse (RN) to intervene? a. The LPN/LVN has the patient to sit in a chair for 2 hours. b. The LPN/LVN gives the prescribed aspirin after breakfast. c. The LPN/LVN assists the patient to walk 40 feet in the hallway. d. The LPN/LVN places the patient in Fowler's position for meals.

ANS A The patient should avoid sitting for long periods because of the increased stress on the suture line caused by leg edema and because of the risk for venous thromboembolism (VTE). The other actions by the LPN/LVN are appropriate. DIF: Cognitive Level: Apply (application) REF: 806 OBJ: Special Questions: Delegation TOP: Nursing Process: Implementation MSC: NCLEX: Safe and Effective Care Environment

6. During a visit to a 78-yr-old patient with chronic heart failure, the home care nurse finds that the patient has ankle edema, a 2-kg weight gain over the past 2 days, and complains of "feeling too tired to get out of bed." Based on these data, a correct nursing diagnosis for the patient is a. activity intolerance related to fatigue. b. impaired skin integrity related to edema. c. disturbed body image related to weight gain. d. impaired gas exchange related to dyspnea on exertion.

ANS A The patient's statement supports the diagnosis of activity intolerance. There are no data to support the other diagnoses, although the nurse will need to assess for additional patient problems. DIF: Cognitive Level: Apply (application) REF: 750 TOP: Nursing Process: Diagnosis MSC: NCLEX: Physiological Integrity

Which nursing action should be included in the plan of care after endovascular repair of an abdominal aortic aneurysm? a. Record hourly chest tube drainage. b. Monitor fluid intake and urine output. c. Assess the abdominal incision for redness. d. Teach the patient to plan for a long recovery period.

ANS B Because renal artery occlusion can occur after endovascular repair, the nurse should monitor parameters of renal function such as intake and output. Chest tubes will not be needed for endovascular surgery, the recovery period will be short, and there will not be an abdominal wound. DIF: Cognitive Level: Apply (application) REF: 815 TOP: Nursing Process: Planning MSC: NCLEX: Physiological Integrity

The nurse is admitting a patient newly diagnosed with peripheral artery disease. Which admission order should the nurse question? a. Cilostazol drug therapy b. Omeprazole drug therapy c. Use of treadmill for exercise d. Exercise to the point of discomfort

ANS B Because the antiplatelet effect of clopidogrel is reduced when it is used with omeprazole, the nurse should clarify this order with the health care provider. The other interventions are appropriate for a patient with peripheral artery disease. DIF: Cognitive Level: Apply (application) REF: 805 TOP: Nursing Process: Assessment MSC: NCLEX: Physiological Integrity COMPLETION

While working in the outpatient clinic, the nurse notes that a patient has a history of intermittent claudication. Which statement by the patient would support this information? a. "When I stand too long, my feet start to swell." b. "My legs cramp when I walk more than a block." c. "I get short of breath when I climb a lot of stairs." d. "My fingers hurt when I go outside in cold weather."

ANS B Cramping that is precipitated by a consistent level of exercise is descriptive of intermittent claudication. Finger pain associated with cold weather is typical of Raynaud's phenomenon. Shortness of breath that occurs with exercise is not typical of intermittent claudication, which is reproducible. Swelling associated with prolonged standing is typical of venous disease. DIF: Cognitive Level: Apply (application) REF: 803 TOP: Nursing Process: Assessment MSC: NCLEX: Physiological Integrity

A patient has a 6-cm thoracic aortic aneurysm that was discovered during routine chest x-ray. When obtaining an admission history from the patient, it will be most important for the nurse to ask about a. low back pain. c. abdominal tenderness. b. trouble swallowing. d. changes in bowel habits.

ANS B Difficulty swallowing may occur with a thoracic aneurysm because of pressure on the esophagus. The other symptoms will be important to assess for in patients with abdominal aortic aneurysms. DIF: Cognitive Level: Analyze (analysis) REF: 810 TOP: Nursing Process: Assessment MSC: NCLEX: Physiological Integrity

The health care provider prescribes an infusion of heparin and daily partial thromboplastin time (PTT) testing for a patient with venous thromboembolism (VTE). The nurse will plan to a. decrease the infusion when the PTT value is 65 seconds. b. avoid giving IM medications to prevent localized bleeding. c. have vitamin K available in case reversal of the heparin is needed. d. monitor posterior tibial and dorsalis pedis pulses with the Doppler.

ANS B Intramuscular injections are avoided in patients receiving anticoagulation to prevent hematoma formation and bleeding from the site. A PTT of 65 seconds is within the therapeutic range. Vitamin K is used to reverse warfarin. Pulse quality is not affected by VTE. DIF: Cognitive Level: Apply (application) REF: 823 TOP: Nursing Process: Planning MSC: NCLEX: Physiological Integrity

The nurse who works in the vascular clinic has several patients with venous insufficiency scheduled today. Which patient should the nurse assign to an experienced licensed practical/vocational nurse (LPN/LVN)? a. Patient who has been complaining of increased edema and skin changes in the legs b. Patient who needs wound care for a chronic venous stasis ulcer on the right lower leg c. Patient who has a history of venous thromboembolism and is complaining of dyspnea d. Patient who needs teaching about elastic compression stockings for venous insufficiency

ANS B LPN education and scope of practice includes wound care. The other patients, which require more complex assessments or education, should be managed by the RN. DIF: Cognitive Level: Apply (application) REF: 827 OBJ: Special Questions: Delegation | Special Questions: Multiple Patients TOP: Nursing Process: Planning MSC: NCLEX: Safe and Effective Care Environment

A patient with a venous thromboembolism (VTE) is started on enoxaparin (Lovenox) and warfarin (Coumadin). The patient asks the nurse why two medications are necessary. Which response by the nurse is most accurate? a. "Taking two blood thinners greatly reduces the risk for another clot to form." b. "Enoxaparin will work right away, but warfarin takes several days to begin preventing clots." c. "Enoxaparin will start to dissolve the clot, and warfarin will prevent any more clots from forming." d. "Because of the risk for a blood clot in the lungs, it is important for you to take more than one blood thinner."

ANS B Low molecular weight heparin (LMWH) is used because of the immediate effect on coagulation and discontinued once the international normalized ratio (INR) value indicates that the warfarin has reached a therapeutic level. LMWH has no thrombolytic properties. The use of two anticoagulants is not related to the risk for pulmonary embolism, and two are not necessary to reduce the risk for another VTE. Anticoagulants do not thin the blood. DIF: Cognitive Level: Apply (application) REF: 820 TOP: Nursing Process: Implementation MSC: NCLEX: Physiological Integrity

The nurse has started discharge teaching for a patient who is to continue warfarin (Coumadin) after hospitalization for venous thromboembolism (VTE). The nurse determines that additional teaching is needed when the patient says which of the following? a. "I should get a Medic Alert device stating that I take warfarin." b. "I should reduce the amount of green, leafy vegetables that I eat." c. "I will need routine blood tests to monitor the effects of the warfarin." d. "I will check with my health care provider before I begin any new drugs."

ANS B Patients taking warfarin are taught to follow a consistent diet with regard to foods that are high in vitamin K, such as green, leafy vegetables. The other patient statements are accurate. DIF: Cognitive Level: Apply (application) REF: 820 TOP: Nursing Process: Evaluation MSC: NCLEX: Physiological Integrity

Which instructions should the nurse include in a teaching plan for an older patient newly diagnosed with peripheral artery disease (PAD)? a. "Exercise only if you do not experience any pain." b. "It is very important that you stop smoking cigarettes." c. "Try to keep your legs elevated whenever you are sitting." d. "Put elastic compression stockings on early in the morning."

ANS B Smoking cessation is essential for slowing the progression of PAD to critical limb ischemia and reducing the risk of myocardial infarction and death. Circulation to the legs will decrease if the legs are elevated. Patients with PAD are taught to exercise to the point of feeling pain, rest, and then resume walking. Support hose are not used for patients with PAD. DIF: Cognitive Level: Apply (application) REF: 817 TOP: Nursing Process: Planning MSC: NCLEX: Physiological Integrity

20. After receiving change-of-shift report on four patients admitted to a heart failure unit, which patient should the nurse assess first? a. A patient who reported dizziness after receiving the first dose of captopril b. A patient who is cool and clammy, with new-onset confusion and restlessness c. A patient who has crackles bilaterally in the lung bases and is receiving oxygen. d. A patient who is receiving IV nesiritide (Natrecor) and has a blood pressure of 100/62

ANS B The patient who has "wet-cold" clinical manifestations of heart failure is perfusing inadequately and needs rapid assessment and changes in management. The other patients also should be assessed as quickly as possible but do not have indications of severe decreases in tissue perfusion. DIF: Cognitive Level: Analyze (analysis) REF: 742 OBJ: Special Questions: Prioritization | Special Questions: Multiple Patients TOP: Nursing Process: Assessment MSC: NCLEX: Physiological Integrity

25. After receiving change-of-shift report on a heart failure unit, which patient should the nurse assess first? a. Patient who is taking carvedilol (Coreg) and has a heart rate of 58 b. Patient who is taking digoxin and has a potassium level of 3.1 mEq/L c. Patient who is taking captopril and has a frequent nonproductive cough d. Patient who is taking isosorbide dinitrate/hydralazine (BiDil) and has a headache

ANS B The patient's low potassium level increases the risk for digoxin toxicity and potentially life-threatening dysrhythmias. The nurse should assess the patient for other signs of digoxin toxicity and then notify the health care provider about the potassium level. The other patients also have side effects of their drugs, but their symptoms do not indicate potentially life-threatening complications. DIF: Cognitive Level: Analyze (analysis) REF: 748 OBJ: Special Questions: Prioritization | Special Questions: Multiple Patients TOP: Nursing Process: Assessment MSC: NCLEX: Safe and Effective Care Environment

After receiving change of shift report, which patient admitted to the emergency department should the nurse assess first? a. A 67-yr-old patient who has a gangrenous left foot ulcer with a weak pedal pulse b. A 50-yr-old patient who is complaining of sudden sharp and severe upper back pain c. A 39-yr-old patient who has right calf tenderness, redness, and swelling after a plane ride d. A 58-yr-old patient who is taking anticoagulants for atrial fibrillation and has black stools

ANS B The patient's presentation of sudden sharp and severe upper back pain is consistent with dissecting thoracic aneurysm, which will require the most rapid intervention. The other patients also require rapid intervention but not before the patient with severe pain. DIF: Cognitive Level: Analyze (analysis) REF: 810 OBJ: Special Questions: Prioritization | Special Questions: Multiple Patients TOP: Nursing Process: Assessment MSC: NCLEX: Physiological Integrity

Which actions could the nurse delegate to unlicensed assistive personnel (UAP) who are providing care for a patient who is at risk for venous thromboembolism? a. Monitor for any bleeding after anticoagulation therapy is started. b. Apply sequential compression device whenever the patient is in bed. c. Ask the patient about use of herbal medicines or dietary supplements. d. Instruct the patient to call immediately if any shortness of breath occurs.

ANS B UAP training includes the use of equipment that requires minimal nursing judgment, such as sequential compression devices. Patient assessment and teaching require more education and critical thinking and should be done by the registered nurse (RN). DIF: Cognitive Level: Apply (application) REF: 824 OBJ: Special Questions: Delegation TOP: Nursing Process: Planning MSC: NCLEX: Safe and Effective Care Environment

8. When teaching the patient with newly diagnosed heart failure about a 2000-mg sodium diet, the nurse explains that foods to be restricted include a. canned and frozen fruits. b. yogurt and milk products. c. fresh or frozen vegetables. d. eggs and other high-protein foods.

ANS B Yogurt and milk products (e.g., cheese) naturally contain a significant amount of sodium, and the intake of these should be limited for patients on a diet that limits sodium to 2000 mg daily. The other foods listed have minimal levels of sodium and can be eaten without restriction. DIF: Cognitive Level: Apply (application) REF: 749 TOP: Nursing Process: Implementation MSC: NCLEX: Physiological Integrity

24. A patient who is receiving dobutamine for the treatment of acute decompensated heart failure (ADHF) has the following nursing interventions included in the plan of care. Which action will be most appropriate for the registered nurse (RN) to delegate to an experienced licensed practical/vocational nurse (LPN/LVN)? a. Teach the patient the reasons for remaining on bed rest. b. Change the peripheral IV site according to agency policy. c. Monitor the patient's blood pressure and heart rate every hour. d. Titrate the rate to keep the systolic blood pressure >90 mm Hg.

ANS C An experienced LPN/LVN would be able to monitor BP and heart rate and would know to report significant changes to the RN. Teaching patients, making adjustments to the drip rate for vasoactive drugs, and inserting a new peripheral IV catheter require RN level education and scope of practice. DIF: Cognitive Level: Apply (application) REF: 745 OBJ: Special Questions: Delegation TOP: Nursing Process: Planning MSC: NCLEX: Safe and Effective Care Environment

13. Which diagnostic test will be most useful to the nurse in determining whether a patient admitted with acute shortness of breath has heart failure? a. Serum troponin b. Arterial blood gases c. B-type natriuretic peptide d. 12-lead electrocardiogram

ANS C B-type natriuretic peptide (BNP) is secreted when ventricular pressures increase, as they do with heart failure. Elevated BNP indicates a probable or very probable diagnosis of heart failure. A 12-lead electrocardiogram, arterial blood gases, and troponin may also be used in determining the causes or effects of heart failure but are not as clearly diagnostic of heart failure as BNP. DIF: Cognitive Level: Analyze (analysis) REF: 740 TOP: Nursing Process: Assessment MSC: NCLEX: Physiological Integrity

The nurse performing an assessment of a patient who has chronic peripheral artery disease (PAD) of the legs and an ulcer on the right second toe would expect to find a. dilated superficial veins. b. swollen, dry, scaly ankles. c. prolonged capillary refill in all the toes. d. serosanguineous drainage from the ulcer.

ANS C Capillary refill is prolonged in PAD because of the slower and decreased blood flow to the periphery. The other listed clinical manifestations are consistent with chronic venous disease. DIF: Cognitive Level: Apply (application) REF: 807 TOP: Nursing Process: Assessment MSC: NCLEX: Physiological Integrity

15. A patient with heart failure has a new order for captopril 12.5 mg PO. After giving the first dose and teaching the patient about the drug, which statement by the patient indicates that teaching has been effective? a. "I will be sure to take the medication with food." b. "I will need to eat more potassium-rich foods in my diet." c. "I will call for help when I need to get up to use the bathroom." d. "I will expect to feel more short of breath for the next few days."

ANS C Captopril can cause hypotension, especially after the initial dose, so it is important that the patient not get up out of bed without assistance until the nurse has had a chance to evaluate the effect of the first dose. The angiotensin-converting enzyme (ACE) inhibitors are potassium sparing, and the nurse should not teach the patient to purposely increase sources of dietary potassium. Increased shortness of breath is expected with the initiation of -adrenergic blocker therapy for heart failure, not for ACE inhibitor therapy. ACE inhibitors are best absorbed when taken an hour before eating. DIF: Cognitive Level: Apply (application) REF: 747 TOP: Nursing Process: Evaluation MSC: NCLEX: Physiological Integrity

A 46-yr-old service-counter worker undergoes sclerotherapy for treatment of superficial varicose veins at an outpatient center. Which instructions should the nurse provide to the patient before discharge? a. Sitting at the work counter, rather than standing, is recommended. b. Exercise, such as walking or jogging, can cause recurrence of varicosities. c. Elastic compression stockings should be applied before getting out of bed. d. Taking an aspirin daily will help prevent clots from forming around venous valves.

ANS C Elastic compression stockings are applied with the legs elevated to reduce pressure in the lower legs. Walking is recommended to prevent recurrent varicosities. Sitting and standing are both risk factors for varicose veins and venous insufficiency. An aspirin a day is not adequate to prevent venous thrombosis and would not be recommended for a patient who had just had sclerotherapy. DIF: Cognitive Level: Apply (application) REF: 825 TOP: Nursing Process: Implementation MSC: NCLEX: Physiological Integrity

22. A patient has recently started on digoxin (Lanoxin) in addition to furosemide (Lasix) and captopril for the management of heart failure. Which assessment finding by the home health nurse is a priority to communicate to the health care provider? a. Presence of 1+ to 2+ edema in the feet and ankles b. Palpable liver edge 2 cm below the ribs on the right side c. Serum potassium level 3.0 mEq/L after 1 week of therapy d. Weight increase from 120 pounds to 122 pounds over 3 days

ANS C Hypokalemia can predispose the patient to life-threatening dysrhythmias (e.g., premature ventricular contractions) and potentiate the actions of digoxin. Hypokalemia also increases the risk for digoxin toxicity, which can also cause life-threatening dysrhythmias. The other data indicate that the patient's heart failure requires more effective therapies, but they do not require nursing action as rapidly as the low serum potassium level. DIF: Cognitive Level: Analyze (analysis) REF: 748 OBJ: Special Questions: Prioritization TOP: Nursing Process: Assessment MSC: NCLEX: Physiological Integrity

12. A 53-yr-old patient with stage D heart failure and type 2 diabetes asks the nurse whether heart transplant is a possible therapy. Which response by the nurse is most accurate? a. "Your heart failure has not reached the end stage yet." b. "You could not manage the multiple complications of that surgery." c. "The suitability of a heart transplant for you depends on many factors." d. "Because you have diabetes, you would not be a heart transplant candidate."

ANS C Indications for a heart transplant include end-stage heart failure (stage D), but other factors such as coping skills, family support, and patient motivation to follow the rigorous posttransplant regimen are also considered. Patients with diabetes who have well-controlled blood glucose levels may be candidates for heart transplant. Although heart transplants can be associated with many complications, there are no data to suggest that the patient could not manage the care. DIF: Cognitive Level: Apply (application) REF: 753 TOP: Nursing Process: Implementation MSC: NCLEX: Physiological Integrity

When caring for a patient on the first postoperative day after an abdominal aortic aneurysm repair, which assessment finding is most important for the nurse to communicate to the health care provider? a. Presence of flatus c. Maroon-colored liquid stool b. Hypoactive bowel sounds d. Abdominal pain with palpation

ANS C Loose, bloody (maroon colored) stools at this time may indicate intestinal ischemia or infarction and should be reported immediately because the patient may need an emergency bowel resection. The other findings are normal on the first postoperative day after abdominal surgery. DIF: Cognitive Level: Analyze (analysis) REF: 813 OBJ: Special Questions: Prioritization TOP: Nursing Process: Assessment MSC: NCLEX: Physiological Integrity

9. The nurse plans discharge teaching for a patient with chronic heart failure who has prescriptions for digoxin (Lanoxin) and hydrochlorothiazide. Appropriate instructions for the patient include a. limit dietary sources of potassium. b. take the hydrochlorothiazide before bedtime. c. notify the health care provider if nausea develops. d. take the digoxin if the pulse is below 60 beats/min.

ANS C Nausea is an indication of digoxin toxicity and should be reported so that the provider can assess the patient for toxicity and adjust the digoxin dose, if necessary. The patient will need to include potassium-containing foods in the diet to avoid hypokalemia. Patients should be taught to check their pulse daily before taking the digoxin and if the pulse is less than 60 beats/min, to call their provider before taking the digoxin. Diuretics should be taken early in the day to avoid sleep disruption. DIF: Cognitive Level: Apply (application) REF: 748 TOP: Nursing Process: Planning MSC: NCLEX: Physiological Integrity

Which assessment finding for a patient who has been admitted with a right calf venous thromboembolism (VTE) requires immediate action by the nurse? a. Erythema of right lower leg c. New onset shortness of breath b. Complaint of right calf pain d. Temperature of 100.4°F (38°C)

ANS C New onset dyspnea suggests a pulmonary embolus, which will require rapid actions such as O2 administration and notification of the health care provider. The other findings are typical of VTE. DIF: Cognitive Level: Analyze (analysis) REF: 824 OBJ: Special Questions: Prioritization TOP: Nursing Process: Planning MSC: NCLEX: Physiological Integrity

5. A patient who has chronic heart failure tells the nurse, "I was fine when I went to bed, but I woke up in the middle of the night feeling like I was suffocating!" The nurse will document this assessment finding as a. orthopnea. b. pulsus alternans. c. paroxysmal nocturnal dyspnea. d. acute bilateral pleural effusion.

ANS C Paroxysmal nocturnal dyspnea is caused by the reabsorption of fluid from dependent body areas when the patient is sleeping and is characterized by waking up suddenly with the feeling of suffocation. Pulsus alternans is the alteration of strong and weak peripheral pulses during palpation. Orthopnea indicates that the patient is unable to lie flat because of dyspnea. Pleural effusions develop over a longer time period. DIF: Cognitive Level: Understand (comprehension) REF: 742 TOP: Nursing Process: Assessment MSC: NCLEX: Physiological Integrity

A young adult patient tells the health care provider about experiencing cold, numb fingers when running during the winter, and Raynaud's phenomenon is suspected. The nurse will anticipate teaching the patient about tests for a. hyperglycemia. c. autoimmune disorders. b. hyperlipidemia. d. coronary artery disease.

ANS C Secondary Raynaud's phenomenon may occur in conjunction with autoimmune diseases such as rheumatoid arthritis. Patients should be screened for autoimmune disorders. Raynaud's phenomenon is not associated with hyperlipidemia, hyperglycemia, or coronary artery disease. DIF: Cognitive Level: Apply (application) REF: 809 TOP: Nursing Process: Planning MSC: NCLEX: Physiological Integrity

4. IV sodium nitroprusside is ordered for a patient with acute pulmonary edema. During the first hours of administration, the nurse will need to titrate the nitroprusside rate down if the patient develops a. ventricular ectopy. b. a dry, hacking cough. c. a systolic BP below 90 mm Hg. d. a heart rate below 50 beats/min.

ANS C Sodium nitroprusside is a potent vasodilator and the major adverse effect is severe hypotension. Coughing and bradycardia are not adverse effects of this medication. Nitroprusside does not cause increased ventricular ectopy. DIF: Cognitive Level: Apply (application) REF: 745 TOP: Nursing Process: Evaluation MSC: NCLEX: Physiological Integrity

2. Which nursing intervention will be most effective when assisting the patient with coronary artery disease (CAD) to make appropriate dietary changes? a. Give the patient a list of low-sodium, low-cholesterol foods that should be included in the diet. b. Emphasize the increased risk for heart problems unless the patient makes the dietary changes. c. Help the patient modify favorite high-fat recipes by using monosaturated oils when possible. d. Inform the patient that a diet containing no saturated fat and minimal salt will be necessary.

ANS: C Lifestyle changes are more likely to be successful when consideration is given to the patient's values and preferences. The highest percentage of calories from fat should come from monosaturated fats. Although low-sodium and low-cholesterol foods are appropriate, providing the patient with a list alone is not likely to be successful in making dietary changes. Completely removing saturated fat from the diet is not a realistic expectation. Up to 7% of calories in the therapeutic lifestyle changes (TLC) diet can come from saturated fat. Telling the patient about the increased risk without assisting further with strategies for dietary change is unlikely to be successful.

14. A 21-yr-old woman is scheduled for percutaneous transluminal balloon valvuloplasty to treat mitral stenosis. Which information should the nurse include when explaining the advantages of valvuloplasty over valve replacement to the patient? a. Biologic valves will require immunosuppressive drugs after surgery. b. Mechanical mitral valves need to be replaced sooner than biologic valves. c. Lifelong anticoagulant therapy is needed after mechanical valve replacement. d. Ongoing cardiac care by a health care provider is not necessary after valvuloplasty.

ANS: C Long-term anticoagulation therapy is needed after mechanical valve replacement, and this would restrict decisions about career and childbearing in this patient. Mechanical valves are durable and last longer than biologic valves. All valve repair procedures are palliative, not curative, and require lifelong health care. Biologic valves do not activate the immune system and immunosuppressive therapy is not needed. DIF: Cognitive Level: Apply (application) REF: 795 TOP: Nursing Process: Implementation MSC: NCLEX: Physiological Integrity

A patient who has chronic heart failure tells the nurse, "I was fine when I went to bed, but I woke up in the middle of the night feeling like I was suffocating!" The nurse will document this assessment finding as a. orthopnea. b. pulsus alternans. c. paroxysmal nocturnal dyspnea. d. acute bilateral pleural effusion.

ANS: C Paroxysmal nocturnal dyspnea is caused by the reabsorption of fluid from dependent body areas when the patient is sleeping and is characterized by waking up suddenly with the feeling of suffocation. Pulsus alternans is the alternation of strong and weak peripheral pulses during palpation. Orthopnea indicates that the patient is unable to lie flat because of dyspnea. Pleural effusions develop over a longer time period

11. The home health nurse is visiting a 30-yr-old patient recovering from rheumatic fever without carditis. The nurse establishes the nursing diagnosis of ineffective health maintenance related to lack of knowledge regarding long-term management of rheumatic fever when the patient makes which statement? a. "I will need prophylactic antibiotic therapy for 5 years." b. "I can take aspirin or ibuprofen (Motrin) to relieve my joint pain." c. "I will be immune to future episodes of rheumatic fever after this infection." d. "I should call the health care provider if I am fatigued or have difficulty breathing."

ANS: C Patients with a history of rheumatic fever are more susceptible to a second episode. Patients with rheumatic fever without carditis require prophylaxis until age 20 years and for a minimum of 5 years. The other patient statements are correct and would not support the nursing diagnosis of ineffective health maintenance. DIF: Cognitive Level: Apply (application) REF: 790 TOP: Nursing Process: Diagnosis MSC: NCLEX: Physiological Integrity

Which information will the nurse include when teaching a patient who is scheduled for a radiofrequency catheter ablation for treatment of atrial flutter? a. The procedure will prevent or minimize the risk for sudden cardiac death. b. The procedure will use cold therapy to stop the formation of the flutter waves. c. The procedure will use electrical energy to destroy areas of the conduction system. d. The procedure will stimulate the growth of new conduction pathways between the atria.

ANS: C Radiofrequency catheter ablation therapy uses electrical energy to "burn" or ablate areas of the conduction system as definitive treatment of atrial flutter (i.e., restore normal sinus rhythm) and tachydysrhythmias. All other statements regarding the procedure are incorrect

IV sodium nitroprusside (Nipride) is ordered for a patient with acute pulmonary edema. During the first hours of administration, the nurse will need to titrate the nitroprusside rate if the patient develops a. ventricular ectopy. b. a dry, hacking cough. c. a systolic BP <90 mm Hg. d. a heart rate <50 beats/minute.

ANS: C Sodium nitroprusside is a potent vasodilator, and the major adverse effect is severe hypotension. Coughing and bradycardia are not adverse effects of this medication. Nitroprusside does not cause increased ventricular ectopy

The nurse working on the heart failure unit knows that teaching an older female patient with newly diagnosed heart failure is effective when the patient states that a. she will take furosemide (Lasix) every day at bedtime. b. the nitroglycerin patch is applied when any chest pain develops. c. she will call the clinic if her weight goes from 124 to 128 pounds in a week. d. an additional pillow can help her sleep if she is feeling short of breath at night.

ANS: C Teaching for a patient with heart failure includes information about the need to weigh daily and notify the health care provider about an increase of 3 pounds in 2 days or 3 to 5 pounds in a week. Nitroglycerin patches are used primarily to reduce preload (not to prevent chest pain) in patients with heart failure and should be used daily, not on an "as needed" basis. Diuretics should be taken earlier in the day to avoid nocturia and sleep disturbance. The patient should call the clinic if increased orthopnea develops, rather than just compensating by further elevating the head of the bed

A patient with chronic heart failure who is taking a diuretic and an angiotensin-converting enzyme (ACE) inhibitor and who is on a low-sodium diet tells the home health nurse about a 5-pound weight gain in the last 3 days. The nurse's priority action will be to a. have the patient recall the dietary intake for the last 3 days. b. ask the patient about the use of the prescribed medications. c. assess the patient for clinical manifestations of acute heart failure. d. teach the patient about the importance of restricting dietary sodium.

ANS: C The 5-pound weight gain over 3 days indicates that the patient's chronic heart failure may be worsening. It is important that the patient be assessed immediately for other clinical manifestations of decompensation, such as lung crackles. A dietary recall to detect hidden sodium in the diet, reinforcement of sodium restrictions, and assessment of medication compliance may be appropriate interventions but are not the first nursing actions indicated.

25. Which assessment finding obtained by the nurse when assessing a patient with acute pericarditis should be reported immediately to the health care provider? a. Pulsus paradoxus 8 mm Hg b. Blood pressure (BP) of 168/94 mm Hg c. Jugular venous distention (JVD) to jaw level d. Level 6 (0 to 10 scale) chest pain with a deep breath

ANS: C The JVD indicates that the patient may have developed cardiac tamponade and may need rapid intervention to maintain adequate cardiac output. Hypertension would not be associated with complications of pericarditis, and the BP is not high enough to indicate that there is any immediate need to call the health care provider. A pulsus paradoxus of 8 mm Hg is normal. Level 6/10 chest pain should be treated but is not unusual with pericarditis. DIF: Cognitive Level: Analyze (analysis) REF: 787 OBJ: Special Questions: Prioritization TOP: Nursing Process: Assessment MSC: NCLEX: Physiological Integrity

16. During the administration of the thrombolytic agent to a patient with an acute myocardial infarction (AMI), the nurse should stop the drug infusion if the patient experiences a. bleeding from the gums. b. increase in blood pressure. c. a decrease in level of consciousness. d. a nonsustained episode of ventricular tachycardia.

ANS: C The change in level of consciousness indicates that the patient may be experiencing intracranial bleeding, a possible complication of thrombolytic therapy. Some bleeding of the gums is an expected side effect of the therapy but not an indication to stop infusion of the thrombolytic medication. A decrease in blood pressure could indicate internal bleeding. A nonsustained episode of ventricular tachycardia is a common reperfusion dysrhythmia and may indicate that the therapy is effective.

Which topic will the nurse plan to include in discharge teaching for a patient with systolic heart failure and an ejection fraction of 33%? a. Need to begin an aerobic exercise program several times weekly b. Use of salt substitutes to replace table salt when cooking and at the table c. Benefits and side effects of angiotensin-converting enzyme (ACE) inhibitors d. Importance of making an annual appointment with the primary care provider

ANS: C The core measures for the treatment of heart failure established by The Joint Commission indicate that patients with an ejection fraction (EF) <40% receive an ACE inhibitor to decrease the progression of heart failure. Aerobic exercise may not be appropriate for a patient with this level of heart failure, salt substitutes are not usually recommended because of the risk of hyperkalemia, and the patient will need to see the primary care provider more frequently than annually

35. The nurse obtains the following data when assessing a patient who experienced an ST-segment-elevation myocardial infarction (STEMI) 2 days previously. Which information is most important to report to the health care provider? a. The troponin level is elevated. b. The patient denies ever having a heart attack. c. Bilateral crackles are auscultated in the mid-lower lobes. d. The patient has occasional premature atrial contractions (PACs).

ANS: C The crackles indicate that the patient may be developing heart failure, a possible complication of myocardial infarction (MI). The health care provider may need to order medications such as diuretics or angiotensin-converting enzyme (ACE) inhibitors for the patient. Elevation in troponin level at this time is expected. PACs are not life-threatening dysrhythmias. Denial is a common response in the immediate period after the MI.

While admitting an 82-year-old with acute decompensated heart failure to the hospital, the nurse learns that the patient lives alone and sometimes confuses the "water pill" with the "heart pill." When planning for the patient's discharge the nurse will facilitate a a. consult with a psychologist. b. transfer to a long-term care facility. c. referral to a home health care agency. d. arrangements for around-the-clock care.

ANS: C The data about the patient suggest that assistance in developing a system for taking medications correctly at home is needed. A home health nurse will assess the patient's home situation and help the patient develop a method for taking the two medications as directed. There is no evidence that the patient requires services such as a psychologist consult, long-term care, or around-the-clock home care

5. After the nurse has finished teaching a patient about the use of sublingual nitroglycerin (Nitrostat), which patient statement indicates that the teaching has been effective? a. "I can expect some nausea as a side effect of nitroglycerin." b. "I should only take the nitroglycerin if I start to have chest pain." c. "I will call an ambulance if I still have pain after taking 3 nitroglycerin 5 minutes apart." d. "Nitroglycerin helps prevent a clot from forming and blocking blood flow to my heart."

ANS: C The emergency medical services (EMS) system should be activated when chest pain or other symptoms are not completely relieved after 3 sublingual nitroglycerin tablets taken 5 minutes apart. Nitroglycerin can be taken to prevent chest pain or other symptoms from developing (e.g., before intercourse). Gastric upset (e.g., nausea) is not an expected side effect of nitroglycerin. Nitroglycerin does not impact the underlying pathophysiology of coronary artery atherosclerosis.

While assessing a 68-year-old with ascites, the nurse also notes jugular venous distention (JVD) with the head of the patient's bed elevated 45 degrees. The nurse knows this finding indicates a. decreased fluid volume. b. jugular vein atherosclerosis. c. increased right atrial pressure. d. incompetent jugular vein valves.

ANS: C The jugular veins empty into the superior vena cava and then into the right atrium, so JVD with the patient sitting at a 45-degree angle reflects increased right atrial pressure. JVD is an indicator of excessive fluid volume (increased preload), not decreased fluid volume. JVD is not caused by incompetent jugular vein valves or atherosclerosis

42. After reviewing information shown in the accompanying figure from the medical records of a 43-year-old, which risk factor modification for coronary artery disease should the nurse include in patient teaching? a. Importance of daily physical activity b. Effect of weight loss on blood pressure c. Dietary changes to improve lipid levels d. Ongoing cardiac risk associated with history of tobacco use

ANS: C The patient has an elevated low-density lipoprotein (LDL) cholesterol and low high-density lipoprotein (HDL) cholesterol, which will increase the risk of coronary artery disease. Although the blood pressure is in the prehypertensive range, the patient's waist circumference and body mass index (BMI) indicate an appropriate body weight. The risk for coronary artery disease a year after quitting smoking is the same as a nonsmoker. The patient's occupation indicates that daily activity is at the levels suggested by national guidelines.

18. In preparation for discharge, the nurse teaches a patient with chronic stable angina how to use the prescribed short-acting and long-acting nitrates. Which patient statement indicates that the teaching has been effective? a. "I will check my pulse rate before I take any nitroglycerin tablets." b. "I will put the nitroglycerin patch on as soon as I get any chest pain." c. "I will stop what I am doing and sit down before I put the nitroglycerin under my tongue." d. "I will be sure to remove the nitroglycerin patch before taking any sublingual nitroglycerin."

ANS: C The patient should sit down before taking the nitroglycerin to decrease cardiac workload and prevent orthostatic hypotension. Transdermal nitrates are used prophylactically rather than to treat acute pain and can be used concurrently with sublingual nitroglycerin. Although the nurse should check blood pressure before giving nitroglycerin, patients do not need to check the pulse rate before taking nitrates.

An outpatient who has chronic heart failure returns to the clinic after 2 weeks of therapy with metoprolol (Toprol XL). Which assessment finding is most important for the nurse to report to the health care provider? a. 2+ pedal edema b. Heart rate of 56 beats/minute c. Blood pressure (BP) of 88/42 mm Hg d. Complaints of fatigue

ANS: C The patient's BP indicates that the dose of metoprolol may need to be decreased because of hypotension. Bradycardia is a frequent adverse effect of b-adrenergic blockade, but the rate of 56 is not unusual with â-adrenergic blocker therapy. b-Adrenergic blockade initially will worsen symptoms of heart failure in many patients, and patients should be taught that some increase in symptoms, such as fatigue and edema, is expected during the initiation of therapy with this class of drugs

17. A patient is recovering from a myocardial infarction (MI) and develops chest pain on day 3 that increases when taking a deep breath and is relieved by leaning forward. Which action should the nurse take next? a. Assess the feet for pedal edema. b. Palpate the radial pulses bilaterally. c. Auscultate for a pericardial friction rub. d. Check the heart monitor for dysrhythmias.

ANS: C The patient's symptoms are consistent with the development of pericarditis, a possible complication of MI. The other assessments listed are not consistent with the description of the patient's symptoms.

13. When caring for a patient with mitral valve stenosis, it is most important that the nurse assess for a. diastolic murmur. c. shortness of breath on exertion. b. peripheral edema. d. right upper quadrant tenderness.

ANS: C The pressure gradient changes in mitral stenosis lead to fluid backup into the lungs, resulting in hypoxemia and dyspnea. The other findings also may be associated with mitral valve disease but are not indicators of possible hypoxemia, which is a priority. DIF: Cognitive Level: Analyze (analysis) REF: 791 TOP: Nursing Process: Assessment MSC: NCLEX: Physiological Integrity

41. A patient with diabetes mellitus and chronic stable angina has a new order for captopril (Capoten). The nurse should teach the patient that the primary purpose of captopril is to a. lower heart rate. b. control blood glucose levels. c. prevent changes in heart muscle. d. reduce the frequency of chest pain.

ANS: C The purpose for angiotensin-converting enzyme (ACE) inhibitors in patients with chronic stable angina who are at high risk for a cardiac event is to decrease ventricular remodeling. ACE inhibitors do not directly impact angina frequency, blood glucose, or heart rate.

A patient with a history of chronic heart failure is admitted to the emergency department (ED) with severe dyspnea and a dry, hacking cough. Which action should the nurse do first? a. Auscultate the abdomen. b. Check the capillary refill. c. Auscultate the breath sounds. d. Assess the level of orientation.

ANS: C This patient's severe dyspnea and cough indicate that acute decompensated heart failure (ADHF) is occurring. ADHF usually manifests as pulmonary edema, which should be detected and treated immediately to prevent ongoing hypoxemia and cardiac/respiratory arrest. The other assessments will provide useful data about the patient's volume status and also should be accomplished rapidly, but detection (and treatment) of pulmonary complications is the priority

14. A patient with ST-segment elevation in three contiguous electrocardiographic (ECG) leads is admitted to the emergency department (ED) and diagnosed as having an ST-segment-elevation myocardial infarction (STEMI). Which question should the nurse ask to determine whether the patient is a candidate for thrombolytic therapy? a. "Do you have any allergies?" b. "Do you take aspirin on a daily basis?" c. "What time did your chest pain begin?" d. "Can you rate your chest pain using a 0 to 10 scale?"

ANS: C Thrombolytic therapy should be started within 6 hours of the onset of the myocardial infarction (MI), so the time at which the chest pain started is a major determinant of the appropriateness of this treatment. The other information will also be needed, but it will not be a factor in the decision about thrombolytic therapy.

Which nursing action can the registered nurse (RN) delegate to experienced unlicensed assistive personnel (UAP) working as a telemetry technician on the cardiac care unit? a. Decide whether a patient's heart rate of 116 requires urgent treatment. b. Monitor a patient's level of consciousness during synchronized cardioversion. c. Observe cardiac rhythms for multiple patients who have telemetry monitoring. d. Select the best lead for monitoring a patient admitted with acute coronary syndrome.

ANS: C UAP serving as telemetry technicians can monitor cardiac rhythms for individuals or groups of patients. Nursing actions such as assessment and choice of the most appropriate lead based on ST segment elevation location require RN-level education and scope of practice

15. While caring for a 23-yr-old patient with mitral valve prolapse (MVP) without valvular regurgitation, the nurse determines that discharge teaching has been effective when the patient states that it will be necessary to a. take antibiotics before any dental appointments. b. limit physical activity to avoid stressing the heart. c. avoid over-the-counter (OTC) drugs that contain stimulants. d. take an aspirin a day to prevent clots from forming on the valve.

ANS: C Use of stimulant drugs should be avoided by patients with MVP because they may exacerbate symptoms. Daily aspirin and restricted physical activity are not needed by patients with mild MVP. Antibiotic prophylaxis is needed for patients with MVP with regurgitation but will not be necessary for this patient. DIF: Cognitive Level: Apply (application) REF: 792 TOP: Nursing Process: Evaluation MSC: NCLEX: Physiological Integrity

Following an acute myocardial infarction, a previously healthy 63-year-old develops clinical manifestations of heart failure. The nurse anticipates discharge teaching will include information about a. digitalis preparations. b. b-adrenergic blockers. c. calcium channel blockers. d. angiotensin-converting enzyme (ACE) inhibitors.

ANS: D ACE inhibitor therapy is currently recommended to prevent the development of heart failure in patients who have had a myocardial infarction and as a first-line therapy for patients with chronic heart failure. Digoxin therapy for heart failure is no longer considered a first-line measure, and digoxin is added to the treatment protocol when therapy with other medications such as ACE-inhibitors, diuretics, and b-adrenergic blockers is insufficient. Calcium channel blockers are not generally used in the treatment of heart failure. The b-adrenergic blockers are not used as initial therapy for new onset heart failure

21. The nurse is obtaining a health history from a 24-yr-old patient with hypertrophic cardiomyopathy (CMP). Which information obtained by the nurse is most important? a. The patient has a history of a recent upper respiratory infection. b. The patient has a family history of coronary artery disease (CAD). c. The patient reports using cocaine a "couple of times" as a teenager. d. The patient's 29-yr-old brother died from a sudden cardiac arrest.

ANS: D About half of all cases of hypertrophic CMP have a genetic basis, and it is the most common cause of sudden cardiac death in otherwise healthy young people. The information about the patient's brother will be helpful in planning care (e.g., an automatic implantable cardioverter-defibrillator [AICD]) for the patient and in counseling other family members. The patient should be counseled against the use of stimulant drugs, but the limited past history indicates that the patient is not currently at risk for cocaine use. Viral infections and CAD are risk factors for dilated cardiomyopathy but not for hypertrophic CMP. DIF: Cognitive Level: Analyze (analysis) REF: 796 TOP: Nursing Process: Assessment MSC: NCLEX: Physiological Integrity

A patient who has just been admitted with pulmonary edema is scheduled to receive the following medications. Which medication should the nurse question before giving? a. Furosemide (Lasix) 60 mg b. Captopril (Capoten) 25 mg c. Digoxin (Lanoxin) 0.125 mg d. Carvedilol (Coreg) 3.125 mg

ANS: D Although carvedilol is appropriate for the treatment of chronic heart failure, it is not used for patients with acute decompensated heart failure (ADHF) because of the risk of worsening the heart failure. The other medications are appropriate for the patient with ADHF

6. Which statement made by a patient with coronary artery disease after the nurse has completed teaching about therapeutic lifestyle changes (TLC) diet indicates that further teaching is needed? a. "I will switch from whole milk to 1% milk." b. "I like salmon and I will plan to eat it more often." c. "I can have a glass of wine with dinner if I want one." d. "I will miss being able to eat peanut butter sandwiches."

ANS: D Although only 30% of the daily calories should come from fats, most of the fat in the TLC diet should come from monosaturated fats such as are found in nuts, olive oil, and canola oil. The patient can include peanut butter sandwiches as part of the TLC diet. The other patient comments indicate a good understanding of the TLC diet.

1. When developing a teaching plan for a 61-year-old man with the following risk factors for coronary artery disease (CAD), the nurse should focus on the a. family history of coronary artery disease. b. increased risk associated with the patient's gender. c. increased risk of cardiovascular disease as people age. d. elevation of the patient's low-density lipoprotein (LDL) level.

ANS: D Because family history, gender, and age are nonmodifiable risk factors, the nurse should focus on the patient's LDL level. Decreases in LDL will help reduce the patient's risk for developing CAD.

11. Nadolol (Corgard) is prescribed for a patient with chronic stable angina and left ventricular dysfunction. To determine whether the drug is effective, the nurse will monitor for a. decreased blood pressure and heart rate. b. fewer complaints of having cold hands and feet. c. improvement in the strength of the distal pulses. d. the ability to do daily activities without chest pain.

ANS: D Because the medication is ordered to improve the patient's angina, effectiveness is indicated if the patient is able to accomplish daily activities without chest pain. Blood pressure and heart rate may decrease, but these data do not indicate that the goal of decreased angina has been met. The noncardioselective β-adrenergic blockers can cause peripheral vasoconstriction, so the nurse would not expect an improvement in distal pulse quality or skin temperature.

When analyzing the rhythm of a patient's electrocardiogram (ECG), the nurse will need to investigate further upon finding a(n) a. isoelectric ST segment. b. P-R interval of 0.18 second. c. Q-T interval of 0.38 second. d. QRS interval of 0.14 second.

ANS: D Because the normal QRS interval is 0.04 to 0.10 seconds, the patient's QRS interval of 0.14 seconds indicates that the conduction through the ventricular conduction system is prolonged. The P-R interval and Q-T interval are within normal range, and ST segment should be isoelectric (flat).

The nurse is caring for a patient who is receiving IV furosemide (Lasix) and morphine for the treatment of acute decompensated heart failure (ADHF) with severe orthopnea. Which clinical finding is the best indicator that the treatment has been effective? a. Weight loss of 2 pounds in 24 hours b. Hourly urine output greater than 60 mL c. Reduction in patient complaints of chest pain d. Reduced dyspnea with the head of bed at 30 degrees

ANS: D Because the patient's major clinical manifestation of ADHF is orthopnea (caused by the presence of fluid in the alveoli), the best indicator that the medications are effective is a decrease in dyspnea with the head of the bed at 30 degrees. The other assessment data also may indicate that diuresis or improvement in cardiac output has occurred, but are not as specific to evaluating this patient's response

4. Which information given by a patient admitted with chronic stable angina will help the nurse confirm this diagnosis? a. The patient states that the pain "wakes me up at night." b. The patient rates the pain at a level 3 to 5 (0 to 10 scale). c. The patient states that the pain has increased in frequency over the last week. d. The patient states that the pain "goes away" with one sublingual nitroglycerin tablet.

ANS: D Chronic stable angina is typically relieved by rest or nitroglycerin administration. The level of pain is not a consistent indicator of the type of angina. Pain occurring at rest or with increased frequency is typical of unstable angina.

1. The nurse obtains a health history from an older patient with a prosthetic mitral valve who has symptoms of infective endocarditis (IE). Which question by the nurse is most focused on identifying a risk factor for IE? a. "Do you have a history of a heart attack?" b. "Is there a family history of endocarditis?" c. "Have you had any recent immunizations?" d. "Have you had dental work done recently?"

ANS: D Dental procedures place the patient with a prosthetic mitral valve at risk for IE. Myocardial infarction, immunizations, and a family history of endocarditis are not risk factors for IE. DIF: Cognitive Level: Apply (application) REF: 781 TOP: Nursing Process: Assessment MSC: NCLEX: Physiological Integrity

A patient has a normal cardiac rhythm and a heart rate of 72 beats/minute. The nurse determines that the P-R interval is 0.24 seconds. The most appropriate intervention by the nurse would be to a. notify the health care provider immediately. b. give atropine per agency dysrhythmia protocol. c. prepare the patient for temporary pacemaker insertion. d. document the finding and continue to monitor the patient.

ANS: D First-degree atrioventricular (AV) block is asymptomatic and requires ongoing monitoring because it may progress to more serious forms of heart block. The rate is normal, so there is no indication that atropine is needed. Immediate notification of the health care provider about an asymptomatic rhythm is not necessary

Which laboratory result for a patient with multifocal premature ventricular contractions (PVCs) is most important for the nurse to communicate to the health care provider? a. Blood glucose 243 mg/dL b. Serum chloride 92 mEq/L c. Serum sodium 134 mEq/L d. Serum potassium 2.9 mEq/L

ANS: D Hypokalemia increases the risk for ventricular dysrhythmias such as PVCs, ventricular tachycardia, and ventricular fibrillation. The health care provider will need to prescribe a potassium infusion to correct this abnormality. Although the other laboratory values also are abnormal, they are not likely to be the etiology of the patient's PVCs and do not require immediate correction

36. A patient had a non-ST-segment-elevation myocardial infarction (NSTEMI) 3 days ago. Which nursing intervention included in the plan of care is most appropriate for the registered nurse (RN) to delegate to an experienced licensed practical/vocational nurse (LPN/LVN)? a. Evaluation of the patient's response to walking in the hallway b. Completion of the referral form for a home health nurse follow-up c. Education of the patient about the pathophysiology of heart disease d. Reinforcement of teaching about the purpose of prescribed medications

ANS: D LPN/LVN education and scope of practice include reinforcing education that has previously been done by the RN. Evaluating the patient response to exercise after a NSTEMI requires more education and should be done by the RN. Teaching and discharge planning/ documentation are higher level skills that require RN education and scope of practice.

When teaching the patient with newly diagnosed heart failure about a 2000-mg sodium diet, the nurse explains that foods to be restricted include a. canned and frozen fruits. b. fresh or frozen vegetables. c. eggs and other high-protein foods. d. milk, yogurt, and other milk products.

ANS: D Milk and yogurt naturally contain a significant amount of sodium, and intake of these should be limited for patients on a diet that limits sodium to 2000 mg daily. Other milk products, such as processed cheeses, have very high levels of sodium and are not appropriate for a 2000-mg sodium diet. The other foods listed have minimal levels of sodium and can be eaten without restriction

30. Which statement by a patient with restrictive cardiomyopathy indicates that the nurse's discharge teaching about self-management has been effective? a. "I will avoid taking aspirin or other antiinflammatory drugs." b. "I can restart my exercise program that includes hiking and biking." c. "I will need to limit my intake of salt and fluids even in hot weather." d. "I will take antibiotics before my teeth are cleaned at the dental office."

ANS: D Patients with restrictive cardiomyopathy are at risk for infective endocarditis and should use prophylactic antibiotics for any procedure that may cause bacteremia. The other statements indicate a need for more teaching by the nurse. Dehydration and vigorous exercise impair ventricular filling in patients with restrictive cardiomyopathy. There is no need to avoid salt (unless ordered), aspirin, or nonsteroidal antiinflammatory drugs. DIF: Cognitive Level: Apply (application) REF: 799 TOP: Nursing Process: Evaluation MSC: NCLEX: Physiological Integrity

43. After reviewing a patient's history, vital signs, physical assessment, and laboratory data, which information shown in the accompanying figure is most important for the nurse to communicate to the health care provider? a. Q waves on ECG b. Elevated troponin levels c. Fever and hyperglycemia d. Tachypnea and crackles in lungs

ANS: D Pulmonary congestion and tachypnea suggest that the patient may be developing heart failure, a complication of myocardial infarction (MI). Mild fever and hyperglycemia are common after MI because of the inflammatory process that occurs with tissue necrosis. Troponin levels will be elevated for several days after MI. Q waves often develop with ST-segment-elevation MI.

8. The nurse has identified a nursing diagnosis of acute pain related to inflammatory process for a patient with acute pericarditis. An appropriate intervention by the nurse for this problem is to a. teach the patient to take deep, slow breaths to control the pain. b. force fluids to 3000 mL/day to decrease fever and inflammation. c. provide a fresh ice bag every hour for the patient to place on the chest. d. place the patient in Fowler's position, leaning forward on the overbed table.

ANS: D Sitting upright and leaning forward frequently will decrease the pain associated with pericarditis. Forcing fluids will not decrease the inflammation or pain. Taking deep breaths will tend to increase pericardial pain. Ice does not decrease this type of inflammation and pain. DIF: Cognitive Level: Apply (application) REF: 785 TOP: Nursing Process: Implementation MSC: NCLEX: Physiological Integrity

The nurse obtains a rhythm strip on a patient who has had a myocardial infarction and makes the following analysis: no visible P waves, P-R interval not measurable, ventricular rate 162, R-R interval regular, and QRS complex wide and distorted, QRS duration 0.18 second. The nurse interprets the patient's cardiac rhythm as a. atrial flutter. b. sinus tachycardia. c. ventricular fibrillation. d. ventricular tachycardia.

ANS: D The absence of P waves, wide QRS, rate >150 beats/minute, and the regularity of the rhythm indicate ventricular tachycardia. Atrial flutter is usually regular, has a narrow QRS configuration, and has flutter waves present representing atrial activity. Sinus tachycardia has P waves. Ventricular fibrillation is irregular and does not have a consistent QRS duration

24. A patient with hyperlipidemia has a new order for colesevelam (Welchol). Which nursing action is most appropriate when giving the medication? a. Have the patient take this medication with an aspirin. b. Administer the medication at the patient's usual bedtime. c. Have the patient take the colesevelam with a sip of water. d. Give the patient's other medications 2 hours after the colesevelam.

ANS: D The bile acid sequestrants interfere with the absorption of many other drugs, and giving other medications at the same time should be avoided. Taking an aspirin concurrently with the colesevelam may increase the incidence of gastrointestinal side effects such as heartburn. An increased fluid intake is encouraged for patients taking the bile acid sequestrants to reduce the risk for constipation. For maximum effect, colesevelam should be administered with meals.

A patient who was admitted with a myocardial infarction experiences a 45-second episode of ventricular tachycardia, then converts to sinus rhythm with a heart rate of 98 beats/minute. Which of the following actions should the nurse take next? a. Immediately notify the health care provider. b. Document the rhythm and continue to monitor the patient. c. Perform synchronized cardioversion per agency dysrhythmia protocol. d. Prepare to give IV amiodarone (Cordarone) per agency dysrhythmia protocol.

ANS: D The burst of sustained ventricular tachycardia indicates that the patient has significant ventricular irritability, and antidysrhythmic medication administration is needed to prevent further episodes. The nurse should notify the health care provider after the medication is started. Defibrillation is not indicated given that the patient is currently in a sinus rhythm. Documentation and continued monitoring are not adequate responses to this situation

The nurse has received change-of-shift report about the following patients on the progressive care unit. Which patient should the nurse see first? a. A patient who is in a sinus rhythm, rate 98, after having electrical cardioversion 2 hours ago b. A patient with new onset atrial fibrillation, rate 88, who has a first dose of warfarin (Coumadin) due c. A patient with second-degree atrioventricular (AV) block, type 1, rate 60, who is dizzy when ambulating d. A patient whose implantable cardioverter-defibrillator (ICD) fired two times today who has a dose of amiodarone (Cordarone) due

ANS: D The frequent firing of the ICD indicates that the patient's ventricles are very irritable, and the priority is to assess the patient and administer the amiodarone. The other patients may be seen after the amiodarone is administered

A patient's cardiac monitor shows sinus rhythm, rate 64. The P-R interval is 0.18 seconds at 1:00 AM, 0.22 seconds at 2:30 PM, and 0.28 seconds at 4:00 PM. Which action should the nurse take next? a. Place the transcutaneous pacemaker pads on the patient. b. Administer atropine sulfate 1 mg IV per agency dysrhythmia protocol. c. Document the patient's rhythm and assess the patient's response to the rhythm. d. Call the health care provider before giving the next dose of metoprolol (Lopressor).

ANS: D The patient has progressive first-degree atrioventricular (AV) block, and the b-blocker should be held until discussing the medication with the health care provider. Documentation and assessment are appropriate but not fully adequate responses. The patient with first-degree AV block usually is asymptomatic, and a pacemaker is not indicated. Atropine is sometimes used for symptomatic bradycardia, but there is no indication that this patient is symptomatic

A patient develops sinus bradycardia at a rate of 32 beats/minute, has a blood pressure (BP) of 80/42 mm Hg, and is complaining of feeling faint. Which actions should the nurse take next? a. Recheck the heart rhythm and BP in 5 minutes. b. Have the patient perform the Valsalva maneuver. c. Give the scheduled dose of diltiazem (Cardizem). d. Apply the transcutaneous pacemaker (TCP) pads.

ANS: D The patient is experiencing symptomatic bradycardia, and treatment with TCP is appropriate. Continued monitoring of the rhythm and BP is an inadequate response. Calcium channel blockers will further decrease the heart rate, and the diltiazem should be held. The Valsalva maneuver will further decrease the rate

After providing a patient with discharge instructions on the management of a new permanent pacemaker, the nurse knows that teaching has been effective when the patient states a. "I will avoid cooking with a microwave oven or being near one in use." b. "It will be 1 month before I can take a bath or return to my usual activities." c. "I will notify the airlines when I make a reservation that I have a pacemaker." d. "I won't lift the arm on the pacemaker side up very high until I see the doctor."

ANS: D The patient is instructed to avoid lifting the arm on the pacemaker side above the shoulder to avoid displacing the pacemaker leads. The patient should notify airport security about the presence of a pacemaker before going through the metal detector, but there is no need to notify the airlines when making a reservation. Microwave oven use does not affect the pacemaker. The insertion procedure involves minor surgery that will have a short recovery period

20. A patient admitted with acute dyspnea is newly diagnosed with dilated cardiomyopathy. Which information will the nurse plan to teach the patient about managing this disorder? a. A heart transplant should be scheduled as soon as possible. b. Elevating the legs above the heart will help relieve dyspnea. c. Careful compliance with diet and medications will prevent heart failure. d. Notify the health care provider about symptoms such as shortness of breath.

ANS: D The patient should be instructed to notify the health care provider about any worsening of heart failure symptoms. Because dilated cardiomyopathy does not respond well to therapy, even patients with good compliance with therapy may have recurrent episodes of heart failure. Elevation of the legs above the heart will worsen symptoms (although this approach is appropriate for a patient with hypertrophic cardiomyopathy). The patient with terminal or end-stage cardiomyopathy may consider heart transplantation. DIF: Cognitive Level: Apply (application) REF: 800 TOP: Nursing Process: Planning MSC: NCLEX: Physiological Integrity

31. When caring for a patient with acute coronary syndrome who has returned to the coronary care unit after having angioplasty with stent placement, the nurse obtains the following assessment data. Which data indicate the need for immediate action by the nurse? a. Heart rate 102 beats/min b. Pedal pulses 1+ bilaterally c. Blood pressure 103/54 mm Hg d. Chest pain level 7 on a 0 to 10 point scale

ANS: D The patient's chest pain indicates that restenosis of the coronary artery may be occurring and requires immediate actions, such as administration of oxygen and nitroglycerin, by the nurse. The other information indicates a need for ongoing assessments by the nurse.

The nurse needs to quickly estimate the heart rate for a patient with a regular heart rhythm. Which method will be best to use? a. Count the number of large squares in the R-R interval and divide by 300. b. Print a 1-minute electrocardiogram (ECG) strip and count the number of QRS complexes. c. Calculate the number of small squares between one QRS complex and the next and divide into 1500. d. Use the 3-second markers to count the number of QRS complexes in 6 seconds and multiply by 10.

ANS: D This is the quickest way to determine the ventricular rate for a patient with a regular rhythm. All the other methods are accurate, but take longer

38. After receiving change-of-shift report about the following four patients, which patient should the nurse assess first? a. 39-year-old with pericarditis who is complaining of sharp, stabbing chest pain b. 56-year-old with variant angina who is to receive a dose of nifedipine (Procardia) c. 65-year-old who had a myocardial infarction (MI) 4 days ago and is anxious about the planned discharge d. 59-year-old with unstable angina who has just returned to the unit after having a percutaneous coronary intervention (PCI)

ANS: D This patient is at risk for bleeding from the arterial access site for the PCI, so the nurse should assess the patient's blood pressure, pulse, and the access site immediately. The other patients should also be assessed as quickly as possible, but assessment of this patient has the highest priority.

8. A patient who has had chest pain for several hours is admitted with a diagnosis of rule out acute myocardial infarction (AMI). Which laboratory test should the nurse monitor to help determine whether the patient has had an AMI? a. Myoglobin b. Homocysteine c. C-reactive protein d. Cardiac-specific troponin

ANS: D Troponin levels increase about 4 to 6 hours after the onset of myocardial infarction (MI) and are highly specific indicators for MI. Myoglobin is released within 2 hours of MI, but it lacks specificity and its use is limited. The other laboratory data are useful in determining the patient's risk for developing coronary artery disease (CAD) but are not helpful in determining whether an acute MI is in progress.


संबंधित स्टडी सेट्स

Openstax Sociology Midterm Review Chapters 1-10

View Set

NR368 Ch. 57: Management of Patients with Burn Injury

View Set

The Individual Life Insurance Contract Quizzes

View Set

marketing 4th test true or false

View Set

Reproductive System Practice Questions

View Set

Physics Chapter 22, Physics Final

View Set

Ch 4: Human Digestion, Absorption and Transport

View Set

chapter 22 and 23 maternal newborn

View Set